2020 Flashcards
A 51-year-old woman is undergoing free flap breast reconstruction. Following anastomosis, the patient sustains a venous thrombotic event, and the decision is made to flush the flap with tissue plasminogen activator (tPA). Which of the following is the primary mechanism of action of tPA as used in this scenario? A ) Antithrombin III activation B) Fibrinolysis C) Inhibition of platelet aggregation D) Protein C activation E) Prothrombin cleavage
B. During microsurgical procedures, the normal clotting mechanism may disrupt flow at the anastomosis. Multiple medications are available to limit clotting following the failure of an anastomosis. However, only certain medications are fibrinolytic and actively break down clots, whereas others limit the formation of further clots. Tissue plasminogen activator (tPA) is one such fibrinolytic agent, which increases the cleavage of the zymogen, plasminogen, to its active form, plasmin. Plasmin is directly fibrinolytic.
Prothrombin cleavage, to form activated thrombin, is primarily facilitated by factor X and results in increased thrombogenesis. Aspirin is a common drug that inhibits platelet aggregation, but this does not have a fibrinolytic effect and is not the mechanism by which tPA functions. Antithrombin III activation is the main mechanism of action of heparin, which limits multiple points in the thrombosis pathway. This medication is not fibrinolytic. Activated protein C is a powerful anticoagulant that inhibits both factors V and VIII in the coagulation cascade. Use of a recombinant protein C has been used in septic shock, but its benefits remain controversial. tPA does not function by protein C activation.
A 68-year-old woman presents with a 3-cm morpheaform basal cell carcinoma (BCCA) involving the left mid cheek. A photograph is shown. Which of the following is an indication for Mohs micrographic surgery over conventional excision? A) Anatomic location of lesion B) Diagnosis of BCCA C) History of previous melanoma D) Morpheaform subtype E ) Patient age
D. Mohs micrographic surgical technique has demonstrated cure rates of 99% for primary basal cell carcinomas (BCCA) and up to 95% for recurrent BCCAs. In this particular patient, the strongest indication for utilization of the Mohs technique is the more aggressive morpheaform subtype of BCCA. This patient underwent Mohs excision with adjacent tissue transfer reconstruction. Patient age, history of previous melanoma, and the diagnosis of BCCA (without aggressive features) are not indications for Mohs excision.
Other indications for Mohs technique include the following:
1. Recurrent basal cell/squamous cell carcinomas;
2. Locations prone to recurrence—“H-zone” of the face: periorbital, periauricular,
temple, upper lip, nose/nasolabial fold, and chin;
3. Tumors involving critical structures such as the eyelid or lip;
4. Functionally important areas such as the genitals, perianal location, hands, and feet;
5. Tumors arising in sites of previous irradiation therapy;
6. Large tumors (greater than 2 cm);
7. Lesions with ill-defined tumor margins;
8. Histologic aggressive subtype (morpheaform, basosquamous, perineural, and
invasive/poorly-differentiated squamous cell carcinoma);
9. Tumors arising in immunosuppressed patients, such as transplant recipients or
patients with genetic predisposition (e.g. basal cell nevus syndrome, xeroderma pigmentosum).
A 55-year-old man presents for a large abdominal midline hernia repair. A component separation is planned with a posterior approach and a retrorectus mesh placement. Which of the following layers can be divided to provide further release and preserve the innervation to the rectus muscle? A) Anterior rectus sheath B) External oblique C) Internal oblique D) Transversalis fascia E ) Transversus abdominis
The correct response is Option E.
In the posterior component separation approach for ventral hernia repair, transversus abdominis release (TAR) can provide further mobility and preserve the innervation to the rectus muscle. The posterior approach reinforces hernia repair with a sublay mesh placed between the rectus muscle and posterior sheath. The Rives-Stoppa approach is associated with a 3 to 6% recurrence rate. To avoid disruption of the segmental nerves to the rectus, classical dissection was limited medial to the linea semilunaris. This, however, limited
the space and reserved this technique for small- to medium-sized hernias. To extend this dissection laterally for use in larger defects, either the internal oblique or the transversus abdominis muscle can be divided. Division of the internal oblique divides the nerves to the rectus muscle. Division of the transversus abdominis can preserve these nerves. With this technique, the anterior rectus sheath is preserved as well as the external oblique and transversalis fascia.
A 56-year-old man who works at a fertilizer production plant presents to the emergency department with 10% hydrofluoric acid burns to the palmar surface of both hands. He is in exquisite pain. Physical examination shows no other injuries. Which of the following is the most appropriate management of this burn injury?
A) Application of a dilute alkali to neutralize the acid
B) Copious surface irrigation and application of calcium gluconate gel
C) Frequent electrolyte and renal lab analyses due to systemic toxicity
D) Mafenide acetate (Sulfamylon) dressings
E) Pain control and maintenance intravenous fluids
The patient has sustained a hydrofluoric acid burn, an agent used in many industrial and domestic applications. Hydrofluoric acid is a unique acidic chemical agent because it can behave as a strong acid at higher concentrations and can also cause liquefactive necrosis, as alkalis do by the dissociation of fluoride ions into subcutaneous tissues. Fluoride ions combine with calcium resulting in local hyperkalemia, which is believed to be the cause for the “pain out of proportion” examination finding associated with hydrofluoric acid burns.
Application of a dilute alkali is not recommended for the treatment of acidic burns because the resulting reaction can be exothermic, resulting in additional injury as the acid is being neutralized.
For patients with greater than 5% total body surface area (TBSA) exposure to hydrofluoric acid or injury with less than 50% concentration of hydrofluoric acid, systemic toxicity can result, causing electrolyte disturbances and organ dysfunction. This patient’s injury resulted from a low-concentration hydrofluoric acid exposure to a small surface area, making serial laboratory assessment less of a priority during presentation.
Due to the intense pain that hydrofluoric acid burns can cause, pain control will be required for management, but maintenance fluid and pain control alone will not treat the burn injury.
The hallmark of hydrofluoric acid chemical burn injury treatment is skin surface irrigation with copious amounts of water at lower pressure, followed by topical calcium gluconate to bind the fluoride ions before they penetrate into the soft tissues. This will neutralize the burn reaction, bind the fluoride ions, and help with pain control.
Mafenide acetate is an appropriate topical antiseptic for non-chemical burns, but will not neutralize hydrofluoric acid.
A 65-year-old woman presents to the office with an ulcer on the right chest wall. She underwent right-sided mastectomy and adjuvant external beam radiation therapy for advanced breast cancer 5 years ago. Physical examination shows a 2-cm ulcer with surrounding radiation-damaged skin and no signs of acute infection. Which of the following is the most appropriate next step in management?
A) Biopsy of the wound
B) Excision of all radiation-damaged tissue and coverage with vascularized tissue
C) Excision of the ulcer and coverage with vascularized tissue
D) Hyperbaric oxygen therapy
E) Negative pressure therapy
The correct response is Option A.
Radiation causes production of reactive oxygen species, which causes injury to tissues and progenitor cells. Cytokine release results in chronic inflammation and ongoing tissue damage. Radiation therapy can cause soft-tissue ulcerations, osteoradionecrosis, and radiation-induced sarcomas. If a patient presents with a late ulcer after radiotherapy, malignancy needs to be ruled out. A biopsy of the ulcer edge should be performed.
Once malignancy has been ruled out, excision of all radiation-damaged tissue, rather than just the ulcer, will result in more durable reconstructive outcomes. Osteoradionecrosis of the chest wall presents as full-thickness chest wall ulcers and the involved ribs should be resected. The underlying pleura and lung may be adherent and, thus, limited lung resection may need to be performed. Reconstruction is performed with well-vascularized tissue, either local pedicled flaps or free flaps.
Negative pressure therapy utilizes subatmospheric pressure for local wound care. It provides local wound care by controlling exudate and, thus, keeping the wound clean. It is thought to promote wound healing by inducing cellular proliferation and increasing capillary blood flow. Malignancy in the wound is a contraindication to negative pressure therapy. Therefore, if suspected, malignancy should be ruled out prior to initiation of negative pressure therapy.
Hyperbaric oxygen is the administration of 100% oxygen in a pressurized chamber. This results in high tissue concentrations of oxygen, which promote neovascularization and wound healing. Hyperbaric oxygen has been shown to improve healing in soft-tissue radionecrosis and osteoradionecrosis. It can be used as an adjunct, especially when radical excision and reconstruction of radiation damaged tissue is not possible.
A 63-year-old man with a BMI of 35 kg/m2 presents with an incisional hernia. The patient underwent a midline exploratory laparotomy for trauma one year ago. Primary fascial closure was achieved with a running polypropylene suture that was performed at the time of the initial operation. CT scan shows intact rectus muscles, and the hernia defect is measured to be 10 cm at the widest, which is in the supraumbilical region. Which of the following is the most effective treatment to prevent hernia recurrence following repair?
A) Component separation with bridging mesh repair
B) Component separation with overlay mesh repair
C) Component separation with primary fascial closure
D) Component separation with retrorectus mesh repair
E) Primary fascial closure
The correct response is Option D.
Hernia repair is associated with a high rate of recurrence, approaching 20% in many studies. Recurrence rates are lowest when primary fascial closure of the abdominal wall is reinforced with mesh placement as an underlay. Primary fascial closure alone or with component
separation results in a higher recurrence rate than primary fascial closure with mesh reinforcement. In this example, it is unlikely that primary fascial closure would be possible, given a 10-cm hernia defect. With regard to mesh placement, there are multiple planes at which the mesh can be placed. Using a bridging repair, the mesh is used to bridge across a fascial defect and is associated with the highest rates of recurrence. In a retrorectus repair, the mesh is placed deep to the rectus (Rives- Stoppa technique) or below the transversus abdominis (transversus abdominis release technique). This is performed underneath a primary fascial closure. Conversely, in an overlay repair, the mesh is secured superficial to the abdominal wall repair. Retrorectus placement of a mesh is associated with a significantly lower recurrence rate than placement of the mesh in another position.
A 45-year-old man presents for reconstruction of a 6 × 11-mm defect involving the nasal alar margin after excision of basal cell carcinoma utilizing Mohs micrographic surgery. The defect involves the skin, cartilage, and nasal lining just lateral to the nasal soft triangle. Photographs are shown. Which of the following reconstructive options is most appropriate? A) Composite auricular graft B) Dorsal nasal flap C) Full-thickness skin graft D) Nasolabial flap E) Primary closure
Alar rim defects present a challenging reconstructive problem. The primary reconstructive goals are to reestablish structural support, provide nasal lining if necessary, and provide external skin of similar color and texture. Complications of alar rim reconstruction include poor scars, alar notching, nasal obstruction, and narrowing of the nostril. Several choices are available, but a composite graft from the ear will often obtain an excellent cosmetic result. Skin along the alar rim, soft triangle, and columella is quite thin and firmly attached to the lower lateral cartilages. Likewise, skin along the helical rim is firmly attached to the underlying cartilage and useful for replicating the delicate topography of the columella, soft triangle, and nostril margin. Composite grafts are typically harvested from the helical root, but can be harvested from throughout the ear.
Composite cartilage grafts only interface with the recipient bed around the graft’s perimeter. As a result, their size should be limited to defects less than 1.0 to 1.5 cm in maximal diameter. It is recommended that no portion of the graft be greater than 1.0 cm from the wound edge. Additionally, the wound bed should be well vascularized, and the patient should be a non-smoker. Composite cartilage grafts follow a predictable healing pattern: white, blue, and then progressively pink/red as revascularization improves. Perioperative strategies recommended by some authors to increase graft take include steroids, hyperbaric oxygen, and cooling of the graft with iced compresses.
Primary closure would lead to a poor result and distortion of the alar rim. Dorsal nasal flap, nasolabial flap, and a full-thickness skin graft do not provide cartilage support, which would result in likely alar notching and potential collapse. Additionally, the skin from these donor sites would be too thick to replace the thin skin that normally inhabits this location.
An otherwise healthy 35-year-old woman, gravida 3, para 3, presents for abdominoplasty. When combined with non-opioid analgesics and/or NSAIDs, which of the following is the most appropriate pain management for this patient?
A) Administration of epidural anesthetic
B) Infiltration of wound with liposomal bupivacaine
C) Intraoperative dexamethasone administration
D) Intraoperative ketamine infusion
E) Intraoperative lidocaine infusion
Enhanced recovery after surgery (ERAS) protocols are multimodal, multidisciplinary perioperative care pathways designed to achieve rapid recovery after surgery. These pathways include consensus recommendation for postoperative analgesia. In addition, improved postoperative pain control can be obtained with emphasis on the use of procedure-specific pain management. Some procedures have a higher propensity for persistent postoperative pain which generally are neuropathic in origin, ie: mastectomy, thoracotomy, hernia repair, abdominal wall surgeries. The primary goal of an optimal pain therapy is to provide “dynamic” pain relief (pain relief during movement) that would allow early ambulation while reducing opioid consumption. The ideal multimodal analgesic technique would include a local/regional analgesic (wound infiltration or peripheral nerve block) as the principal component because they provide excellent dynamic pain relief. Liposomal bupivacaine (Exparel) allows delivery of bupivacaine for 96 hours with a single local
administration. There can be significant pain relief with the combination of wound infiltration with liposomal bupivacaine acetaminophen and NSAIDs or COX2 inhibitors as multimodal analgesic regimens. Epidural analgesia, dexamethasone, lidocaine and ketamine infusions all have demonstrated postoperative pain relief and reduction in opioid requirements to varying degrees. But local/regional analgesia (wound infiltration) should be used as the first-line analgesic therapy, which should be combined with acetaminophen, NSAIDs or COX1 inhibitors.
A 46-year-old man presents with a midline 18-cm-wide ventral hernia 1 year after undergoing midline exploratory laparotomy for a bowel resection and right end ileostomy. Medical history includes significant weight loss through diet and exercise. His weight has been stable for 2 years. BMI is 29 kg/m . He undergoes bilateral
component separation with biologic mesh bridged between the rectus muscles and concomitant panniculectomy. Which of the following clinical characteristics will most likely increase the likelihood of hernia recurrence?
A) BMI greater than 24.9 kg/m2
B) Bridged biologic mesh hernia closure
C) Concomitant panniculectomy
D) Presence of an end ileostomy
E) Prior abdominal surgery
The correct response is Option B.
The patient presents after significant weight loss with a wide midline ventral hernia, right end ileostomy through his rectus muscle, and an abdominal pannus. Given the 18-cm waist of the hernia defect, he is being counseled that only a bridged repair with a biologic mesh will be possible rather than total muscular coverage for the midline defect. Hernia recurrence is a major problem for patients and can be associated with specific characteristics. When the technique of bilateral component separation and inlay biologic mesh repair is being performed, the most important predictor of recurrence is whether the rectus muscle and fascia will be able to be closed at midline, creating a total submuscular repair, or whether the mesh will be bridged. A bridged repair is associated with a 33% chance of recurrence at 3 years compared to 6.2% for total muscle coverage with fascial closure at midline.
With a BMI of 29 kg/m , the patient remains overweight despite his prior stable weight loss.
Surgical site occurrences are increased in the overweight patient with a 26.4% incidence versus 14.9% in patients with BMI less than 24.9 kg/m2. Similarly, skin dehiscence is significantly increased in the overweight patient (19.3% versus 7.2%), while hernia recurrence rates are not statistically significant (11.4% versus 7.7%). Concomitant panniculectomy was associated with an increase in surgical site occurrences and skin dehiscence, but hernia recurrence rates were not affected.
Similarly, patients with existing ileostomies or stomas complicated by parastomal hernias do have a significantly increased surgical site occurrence rate (34.1% with parastomal andmidline hernia versus 18.7% with midline hernia only) but hernia recurrence rates are not affected. Prior abdominal surgery will be in the clinical history of all incisional hernia patients.
A 6-month-old female infant is referred by the pediatrician for management of a skin lesion on the right parietal scalp that was noticed at birth. The lesion is a 2 × 1-cm yellow plaque that is devoid of hair and has grown in proportion with the child. She is otherwise healthy and is doing well. Which of the following is the most appropriate recommendation for the child’s parents?
A) Biopsy to rule out malignancy
B) CO2 laser therapy prior to puberty
C) Excision due to high risk of malignant transformation
D) MRI to evaluate for brain abnormalities
E ) Continued observation because of anesthetic risk
The correct response is Option E.
These clinical features are typical of nevus sebaceous. They present as yellow-orange flat plaques, occurring most commonly on the scalp (60%) or face (30%). They are usually present at birth but may appear in the first few years of life. They are hamartomas, arising from the pilosebaceous units of the skin. They occur due to mutations in the RAS pathway. Maternal transmission of genetic material from the human papilloma virus to the fetus has been implicated as a causative factor.
Excision of nevus sebaceous is performed because of the cosmetic concerns and risk of secondary tumors. The most common neoplasia is trichoblastoma, which is a benign tumor, although more than 40 types of secondary tumors have been described. The most common malignant tumor is basal cell carcinoma. Initial studies reported the risk of malignant transformation to be
10%, however, more recent studies indicate that this number is 1%. The risk of malignant transformation increases with age; it is extremely rare in childhood and has not been reported in children younger than 5 years of age. The risk of malignant transformation is, thus, very small and in the absence of any morphologic change in the lesion, biopsy is not indicated.
Nevus sebaceous lesions undergo change in appearance during puberty and become thick and verrucous, presumably due to hormonal influence. Most practitioners thus recommend definitive treatment prior to puberty. Surface ablative therapies like electrodessication, curettage, dermabrasion, photodynamic and CO2 laser have been proposed to improve the appearance of these lesions. However, nevus cells can be left behind in the deeper layers, with the risk of developing secondary tumors and potentially making future detection of neoplastic change more difficult.The definitive treatment of nevus sebaceous is full thickness skin excision. In December of 2016, the Food and Drug Administration (FDA) issued a warning that “repeated or lengthy use of general anesthetic and sedation drugs during surgeries or procedures in children younger than three years or in pregnant women during their third trimester may affect the development of children’s brains.” The FDA modified the warning in April of 2017, stating “consideration should be given to delaying potentially elective surgery in young children where medically appropriate.”
Most of the data that lead to these warnings came from animal studies that showed learning and behavioral problems after exposure to anesthetics that block N-methyl-D-aspartate (NMDA) and gamma-aminobutyric acid (GABA). Research in humans is not conclusive, with some studies indicating neurotoxicity with multiple exposures, but not with a single exposure. However, the duration of a “brief exposure” has not been well-defined. There are ongoing studies that will hopefully shed further light on the matter. In view of this, it may be prudent to delay elective procedures in children if this will be not detrimental to the child’s health or final outcome.
Numerous syndromes are associated with nevus sebaceous. These mostly involve the central nervous and ocular system, but can also involve other organs. There does not appear to be a correlation between size of skin lesions and risk of nervous system involvement, but large lesions and centrofacial location have been suggested as having higher risk. Small isolated nevus sebaceous lesions in the absence of any other systemic manifestations do not warrant central nervous system imaging or systemic work up. The vast majority of nevus sebaceous are isolated lesions.
A 42-year-old woman suffers a dog bite injury to her lower lip. Following adequate debridement, the patient is left with a full-thickness, total lower lip defect, up to the lateral commissure bilaterally. The surrounding tissue is uninjured. Which of the following is the most appropriate method of reconstruction?
A) Abbe (lip switch) flap
B) Bernard-Webster (lip-cheek advancement) flap
C) Cervicofacial rotation advancement flap
D) Estlander (lateral lip switch) flap
E) Karapandzic flap
The correct response is Option B.
Complete lower lip defects can result from cancer resection or trauma. Reconstruction of lip defects relies primarily on local flaps, although free tissue transfer may be necessary in total lip reconstructions or if the surrounding tissue is unsuitable for flap transfer. In lower lip defects with a defect size greater than two thirds of the lip, bilateral lip-cheek advancement flaps are required for reconstruction.
Lip switch flaps are useful for one- to two-thirds lip defects that are centrally located, whereas lateral lip-switch flaps can similarly be used to address lateral defects involving the commissure. Although these flaps can be combined with lip-cheek advancement flaps for reconstruction of the lower lip, they are not sufficient alone for total lip reconstruction. Karapandzic flaps can be used for reconstruction of central defects with up to two-thirds of the lip being absent, but cause significant microstomia in lip defects with greater than two- thirds of the lip absent. Cervicofacial rotation advancement flaps are usually used for cheek reconstruction.
A 56-year-old woman is evaluated 6 hours after undergoing bilateral breast reconstruction with a deep inferior epigastric perforator (DIEP) flap. Doppler examination shows strong arterial signals in both flaps. The right breast appears bluish with a capillary refill time of 1 second compared to 3 seconds on the left side. Which of the following is the most appropriate next step?
A) Administration of tissue plasminogen activator
B) Application of leeches
C) Application of nitroglycerin ointment
D) Exploration in the operating room
E) Observation
The correct response is Option D.
The patient described has evidence of venous congestion. The reported incidence of venous congestion in free tissue breast reconstruction ranges from 2 to 20%. Causes include venous thrombosis, inadequate perforator selection, and superficial venous system dominance with lack of sufficient communication to the deep system. Signs of venous compromise include the following: cyanotic/blue color, brisker than normal capillary refill, increased tissue turgor, cooler temperature compared to normal skin (greater than 2 degrees), rapid bleeding of dark blood with pinprick, and absence of continuous venous Doppler signal. The most appropriate course of action in this scenario is emergent exploration in the operating room to assess the vascular pedicle for thrombosis, compression from hematoma, kinking, or superficial system dominance. Flap salvage rate is directly tied to timing of exploration, with higher salvage rates in flaps explored within 6 hours of identification of compromise.
Early recognition and rapid exploration of compromised flaps are the most important factors predicting flap salvage, so observation would be unacceptable. Tissue plasminogen activator is useful if diffuse clotting is suspected within the flap, but should only be given locally within the flap. Leeches can be a useful adjunct postoperatively after employing the other maneuvers described above, but would not resolve the underlying problem in this case. Application of topical nitroglycerin can improve venous congestion in random skin flaps, but has no role in the management of acute microvascular thrombosis.
A 45-year-old woman presents with right breast cancer and is planning a nipple- sparing mastectomy and tissue expander placement. She is specifically interested in a carbon dioxide–based expander. Which of the following is a disadvantage of this device compared with a saline tissue expander? A) Extrusion B) Inability to deflate C) Increase in wound dehiscence D) Increase in wound infection E) Possible device dislocation
The correct response is Option B.
The carbon dioxide-based tissue expander (AeroForm) is a fixed-volume device and has an inability to deflate the expander.
In a prospective, multicenter, randomized controlled trial comparing carbon dioxide–based expanders and saline tissue expanders, there were no statistically significant differences in rates of wound infection, extrusion, device dislocation, or wound dehiscence. Advantages of the carbon dioxide–based expander include a more rapid expansion process and a shorter time to implant exchange. The device is self-contained and patient-controlled, so there are no needles required and possibly fewer physician office visits.
3-year-old child with pectus excavatum deformity is evaluated for surgical correction of the chest wall. The child has experienced mild respiratory insufficiency. Which of the following is the optimal timing of treatment for this patient?
A) Surgical correction between ages 2 and 5
B) Surgical correction between ages 6 and 12
C) Surgical correction between ages 13 and 17
D) Surgical correction at skeletal maturity
The correct response is Option B.
Pectus excavatum is the most common congenital chest wall deformity, occurring in approximately 1 in 400 live births. The condition is more common in males, and there is a positive family history in 30 to 40% of patients. The etiology is thought to be
multifactorial and associated with increased incidence of congenital cardiac abnormalities, connective tissue disorders (e.g., Marfan and Ehlers-Danlos syndromes), and scoliosis. Treatment options have shifted from the traditional open technique involving sternal osteotomy and resection of abnormal costal cartilage to minimally invasive options such as the Nuss procedure and minimally invasive technique for repair of excavatum (MIRPE), which utilizes thoracoscopy and placement of intrathoracic retrosternal support bars to reposition the sternum and allow gradual remodeling over a period of 2 to 4 years. The ideal timing of repair is mid-adolescence, usually between ages 6 and 12.
A 35-year-old woman presents with unilateral swelling that has developed over the past 3 months. She underwent bilateral nipple-sparing mastectomy with immediate implant reconstruction with textured, round silicone gel implants 8 years ago. Ultrasound confirms periprosthetic seroma without any masses. Which of the following is the most appropriate next step in the management of this patient?
A) Core needle biopsy
B) Fine-needle aspiration
C) Implant removal and capsulectomy
D) MRI
E) Positron emission tomography (PET) scan
The correct response is Option B.
The clinical scenario is concerning for breast implant–associated anaplastic large-cell lymphoma (BIA-ALCL). Aspiration of the fluid seen on ultrasonound and
pathologic evaluation is necessary to confirm the diagnosis. Following the National Comprehensive Cancer Network guidelines, initial workup of an enlarged breast should include ultrasound evaluation specifically for a fluid collection, a breast mass, or enlarged regional lymph nodes (axillary, supraclavicular, and internal mammary).
MRI is appropriate for cases where ultrasound is indeterminate or requires further confirmation. This patient does not have an identifiable mass amenable to core biopsy. Positron emission tomography (PET) scan is beneficial in confirmed cases to identify associated masses, chest wall involvement, regional lymphadenopathy, and/or metastasis. Implant removal and capsulectomy is appropriate once the diagnosis is confirmed.
A 76-year-old man sustains a right mandibular body fracture after a mechanical fall. He states that he lives independently and is active. On examination, the patient is noted to be edentulous. A CT scan demonstrates a comminuted fracture mesial to the angle without evidence of any other injuries. Which of the following is the most appropriate treatment of this deformity?
A) Closed reduction and external fixation
B) Intra-oral incision and miniplate fixation along external oblique ridge
C) Maxillomandibular fixation
D) Mechanical soft diet for four weeks
E) Submandibular incision and reconstruction bar fixation
The correct response is Option E.
In edentulous patients, mandibular atrophy can make it difficult to achieve appropriate reduction. Mandibular body fractures are common in these patients following blunt trauma, such as a fall. Regarding the management of a mandible fracture in an edentulous man, open reduction and internal fixation is necessary to provide long-term stability and an accurate restoration of previous anatomy, possibly due to poor osteogenic capacity and ability to load- bear of an atrophic mandible.
Intra-oral and extra-oral incisions are both acceptable exposure methods. Similarly, miniplates or reconstruction bars can both be utilized, although some prefer the added stability of a reconstruction bar. Submandibular incision and reconstruction bar fixation is the most correct initial management, as it involves open reduction and internal fixation of the fracture.
Mechanical soft diet is not appropriate treatment for a comminuted mandibular body fracture and will result in malunion or nonunion. Similarly, the use of maxillomandibular
fixation, with or without dentures, does not accurately reduce the fracture in an edentulous mandible with intrinsic loss of osteogenic potential and can lead to higher complications, such as pneumonia, in elderly patients. Miniplate fixation along the external oblique ridge is a treatment for noncomminuted angle fractures, but is not the appropriate treatment for a comminuted body fracture.
External fixation is usually reserved for injuries with significant soft tissue deficit. It does not provide accurate anatomic alignment.
In women undergoing prosthetic breast reconstruction complicated by an expander/implant infection, which of the following is the most common gram- negative bacteria isolated from cultures? A) Escherichia coli B) Klebsiella C) Proteus D) Pseudomonas E) Serratia
The correct response is Option D.
Tissue expander/implant-based breast reconstruction remains the most common form of reconstruction after mastectomy. One of the most potentially devastating complications of this form of breast reconstruction is an implant infection with need for removal of the expander/implant. The mean reported incidence of implant infection after breast reconstruction is 8%, with a range of 1 to 35%. When cultures are obtained, the most common causative bacteria on microbiology examination are gram-positive organisms (41 to 83%), specifically, Staphylococcus species (56 to 76.5%). Gram-negative bacteria accounted for 15.3 to 28.6%, with Pseudomonas (10.7 to 14%) being the most common gram-negative bacteria present on microbiology examination.
A 32-year-old Caucasian woman presents with multiple (>50) brown lesions on her arms and lower legs. They appear to be in areas of sun exposure. On examination, many of these lesions are well circumscribed, even in color, and less than 5 mm in size. The patient has a family history of melanoma. There are too many lesions to excise. Which of the following findings in one of these lesions would prompt an excisional biopsy? A) Asymmetry B) Clearly demarcated borders C) Dark coloration D) Waxy surface
The correct response is Option A.
In this patient with multiple melanocytic nevi, lesions should be treated with excisional biopsy if there is a high suspicion for melanoma. As there are more than 50 lesions, clearly there are too many to excise. These lesions should be evaluated for asymmetry, border irregularity, variable color, diameter greater than 6 mm, and evolution. Any of these signs in a lesion should lead to an excisional biopsy with a suspicion of melanoma, especially given the patient’s family history.
Lesions with a waxy surface are seborrheic keratoses and commonly found in an elderly population in sun-exposed areas. Dark coloration does not lead to a suspicion of melanoma.
A 23-year-old African-American man presents with a raised thickened scar on his anterior chest that he complains is pruritic and unattractive. It was removed by another provider 4 years earlier and has slowly recurred over the past year. On examination, the lesion extends beyond the initial borders of the scar and is firm and hyper-pigmented. On review of his prior pathology report, which of the following histologic characteristics is most likely?
A) Greater ratio of type III to type I collagen
B) Multitude of myofibroblasts and smooth muscle actin C) Parallel collagen bundles
D ) Thick, wavy, and randomly oriented collagen fibers
The correct response is Option D.
In patients with abnormal or excessive scar tissue formation, treatment and prognosis will be driven by the correct diagnosis of a keloid versus a hypertrophic scar. This patient presents with a recurrent keloid of the chest. His clinical history supports this diagnosis by recurrence after resection, growth extending beyond the original border of the lesion, late recurrence after several years, and continued growth over several years without regression or improvement. Hypertrophic scars are less likely to recur, contained within the original boundaries of the lesion, often regress somewhat within a year, and recur earlier in the postoperative period if they are to recur. Both hypertrophic scars and keloid scars can be pruritic.
Pathologic analysis of keloids reveals more type I collagen than type III collagen, similar to normal skin. Hypertrophic scars will exhibit increased type III collagen and pro-fibrotic collagen cross-linking. Keloid growth is thought to be impacted by cell-signaling between keratinocytes and fibroblasts, but hypertrophic scar production requires an abundance of myofibroblasts expressing smooth muscle actin. While hypertrophic scars have parallel collagen fibrils and bundles, keloids are characterized histologically by thick, randomly oriented collagen fibrils that are not organized into bundles.
A 5-year-old boy presents to the emergency department 4 hours after he sustained an amputation of his left index finger when it was slammed in a door. The parents brought the amputated digit in a plastic bag on ice. The amputation is at the level of the mid proximal phalanx. Which of the following is the most important reason to attempt replantation?
A) The amputation is proximal to the flexor digitorum superficialis insertion
B) The cold ischemia time is less than 6 hours
C) It is the index finger
D) It is a single-digit amputation
E) The patient is a child
The correct response is Option E.
Digital replantation should almost always be attempted in a child, except when the amputated part is severely crushed or there are other life-threatening injuries that preclude surgery. Replantation in children is technically more challenging due to the smaller size of the vessels. However, functional outcomes are more superior than in adults. The replanted parts have better sensory return and can have normal growth. Amputations through joints also exhibit remarkable joint remodeling.
A single digit amputation, especially proximal to the flexor digitorum superficialis (FDS) insertion is considered a contraindication to replantation. Digit replantations proximal to the FDS insertion have a poor range of motion as compared to amputations distal to the FDS insertion. This is, thus, an important landmark when making decisions about amputation versus replantation. Multiple digit amputations are an indication for replantation as the functioning deficit with loss of multiple digits is great. The thumb is responsible for 40% of the function of the hand and should always be replanted, if possible. Even if it is stiff and insensate, a replanted thumb will act as a post for opposition.
Index finger amputations at or proximal to the proximal interphalangeal joint are considered by many to be an indication for amputation. A stiff and painful index finger is likely to be excluded by the patient; amputation will result in better global hand function.
Digits tolerate longer ischemia times than more proximal level amputations, due to absence of muscle. Amputated digits tolerate warm ischemia times of 6 to 12 hours and cold ischemia times of 12 to 24 hours. Digital replantation has been reported with warm ischemia time of 33hours and cold ischemia time of 94 hours. Cold ischemia time is thus not a major consideration in the decision-making process for amputation versus replantation.
A 20-year-old otherwise healthy individual who has a diagnosis of gender dysphoria would like to undergo masculinization of the chest for female-to-male transition. According to the World Professional Association for Transgender Health (WPATH) Standards of Care, which of the following criteria should be fulfilled before the patient can be cleared for such a procedure?
A) The patient should be at least 21 years of age
B) The patient should have completed 12 months of hormone therapy
C) The patient should have health insurance coverage
D) The patient should have lived 12 months in a male gender role
E ) The patient should have one letter of support from a mental health professional
The correct response is Option E.
The World Professional Association for Transgender Health (WPATH) Standards of Care (SOC) lists having one letter of support from a qualified mental health professional as a prerequisite for female-to-male (FTM) chest surgery. As for all of the SOC, the criteria for initiation of surgical treatments for gender dysphoria were developed to promote optimal patient care. While the SOC allow for an individualized approach to best meet a patient’s health care needs, a criterion for all breast/chest and genital surgeries is documentation of persistent gender dysphoria by a qualified mental health professional. For some surgeries, additional criteria include preparation and treatment consisting of feminizing/masculinizing hormone therapy and one year of continuous living in a gender role that is congruent with one’s gender identity. Based on the available evidence and expert clinical consensus, different recommendations are made for different surgeries. For FTM chest surgery, the criteria are as follows:
1. Persistent, well-documented gender dysphoria
2. Capacity to make a fully informed decision and to consent for treatment
3. Age of majority in a given country (if younger, follow the SOC for children and adolescents)
4. If significant medical or mental health concerns are present, they must be reasonably well controlled
5. Hormone therapy is not a prerequisite
6. One referral
For male-to-female breast surgery, it is recommended to have completed 1 year of hormonal therapy, although it is not a specific criterion according to WPATH SOC. For genital surgery, it is recommended to have two referrals, be on 1 year of hormonal therapy, and to have lived in the gender role congruent with their gender identity for at least 1 year. Insurance coverage is not part of any WPATH SOC.
During elevation of the anterolateral thigh flap, the dominant vascular supply most commonly originates from which of the following branches of the lateral circumflex femoral artery? A) Ascending B) Deep C) Descending D ) Oblique E) Transverse
The correct response is Option C.
The anterolateral thigh flap (ALT) has become a workhorse flap throughout the body, particularly in the head and neck. The ALT flap is a musculo-fasciocutaneous flap. The dominant pedicle is the descending branch of the lateral circumflex femoral artery in the majority of flaps. The arterial pedicle along with its two venae comitantes pass obliquely along with the nerve to the vastus lateralis in a groove between the rectus femoris and vastus lateralis muscles. The flap can be raised as a sensate flap by including the anterior branch of the lateral cutaneous nerve of thigh. Perforators to the ALT flap are either septocutaneous (passing between the rectus femoris muscle and the vastus lateralis muscle) or musculocutaneous (passing through the vastus lateralis muscle). Septocutaneous perforators are found in 19.8% of people, while in 1.8% of cases no perforators are found. The majority of the time, the descending branch of the lateral circumflex femoral artery originates from the lateral circumflex femoral artery, but it can have a variable origin. It can arise from the deep femoral artery (6.25 to 13%) or the common femoral artery (1 to 6%).
The dominant perforator supply to the ALT flap:
• Descending branch of the lateral circumflex femoral artery: 57 to 100% • Oblique branch of the lateral circumflex femoral artery: 14 to 43%
• Transverse branch of the lateral circumflex femoral artery: 4 to 35%
• Ascending branch of the lateral circumflex femoral artery: 2.6 to 14.5%
A 10-year-old boy presents with an ear injury sustained after a picture frame fell onto his head. The injury is shown in the photograph. He never lost consciousness and has no other injury. Microsurgical reattachment is not an option. Which of the following is the most appropriate initial treatment? A) Debridement and closure B) Dressing with petroleum gauze C) Immediate flap reconstruction D) Reattach as a composite graft E) Split-thickness skin graft
The correct response is Option D.
While composite grafting of large ear avulsions has a globally poor outcome, the avulsed fragment in this patient is a thin piece of the helical rim that includes only a small piece of the helical rim cartilage. The shape of the defect, minimal cartilage involvement, and the fact that this was a clean injury in a young patient, makes an initial attempt to replace the tissue as a composite graft the best initial option. At worst, the tissue acts to cover the wound until a definitive reconstruction can be planned. At best, the tissue survives to some degree and
salvages some of the delicate and very hard to replace helical rim contour. This patient described in the clinical scenario had 80% survival of the tissue with this technique and required no further reconstruction.
The exposed cartilage is at risk for infection and may dessicate, so a simple dressing change with petroleum is ill advised. Debridement and closure might be possible if more cartilage was removed, but this further compounds the tissue loss. A split-thickness skin graft contracts and may not take well on exposed cartilage. Immediate flap reconstruction is possible, but a flap can always be done at a later time if the composite graft does not survive.
A 45-year-old woman is noted to have a mass in the parotid gland. She has a history of external radiation therapy for a facial keloid scar in her twenties. Which of the following is the most likely diagnosis? A) Acinic cell carcinoma B) Mucoepidermoid carcinoma C) Pleomorphic adenoma D) Squamous cell carcinoma E) Warthin tumor
The correct response is Option C.
Of all adult salivary gland tumors, 75 to 85% occur in the parotid gland, 8 to 15% in the submandibular, and 5 to 8% in the minor salivary glands. Pleomorphic adenoma, or benign mixed tumor, is the most common tumor postirradiation as seen in this clinical scenario.
Mucoepidermoid carcinoma is the most common malignant tumor of the parotid gland. Adenoid cystic tumor is the most common in the submandibular and minor salivary glands. Squamous cell carcinoma in the parotid is usually metastatic from frontotemporal scalp cutaneous skin cancer. It can present as high-grade mucoepidermoid carcinoma. Warthin tumor typically appears in the fifth to seventh decade of life. Metachronous bilaterality is observed in up to 6% of cases and radiation therapy is a well-known predisposing factor.
A 35-year-old man presents after Mohs resection of a basal cell carcinoma at the tip of the nose. The defect is 2.5 cm in diameter and the lower lateral cartilages are exposed. Which of the following is the most appropriate method for closure of the defect? A) Local transposition flap B) Paramedian forehead flap C) Primary closure D) Radial forearm flap E) Split-thickness skin graft
The correct response is Option B.
The closure of Mohs defects of the nose is typically guided by subunit reconstruction and using tissue of like quality and color. Small defects can be attempted primarily, but the tip of the nose will usually require a local transposition flap (bilobed, rhomboid), V-Y advancement, or other similar tissue transposition.
Paramedian flaps are reserved for larger defects such as this 2-cm defect that cannot be accommodated by local transfer, especially in a young and healthy patient. The radial forearm is used in complex or complete nasal reconstruction. A full-thickness skin graft is favored by some for its simplicity and it is often used for flap failures. A split-thickness skin graft will not provide a good match for color or tissue thickness for the tip of the nose.
A 25-year-old healthy man presents with a painful clicking when opening and closing his mouth 12 weeks after being involved in a physical altercation. He is able to chew and open and close his mouth normally, but with discomfort. Anteroposterior x-ray study shows no abnormalities. Which of the following is the most likely source of his discomfort? A) Articular disc subluxation B) Dynamic condylar subluxation C) Early arthritis D) Occult fracture of the condylar head E) Spasm of the lateral pterygoid muscle
The correct response is Option A.
This patient likely has increased mobility of the articular disc. This can occur as a result of acute trauma (as in this case) or chronic trauma, such as bruxism. At this juncture, the disc is reducing with motion, so there is no obstruction to movement. Nevertheless, symptoms can worsen over time and create a closed-lock wherein the patient cannot open his mouth. An MRI and/or ultrasound can help confirm the pathology. With the limited and nonmechanical symptoms (eg, locking), treatment is conservative.
Arthritis is possible but unlikely in a patient of this age, especially without some other reason, such as infection or a history of juvenile rheumatoid arthritis. Fracture of the condylar head is possible, but this should have healed after 2 months and would be asymptomatic. Subluxation of the condylar head would restrict motion, and spasm of the lateral pterygoid can cause temporomandibular joint pain, but subluxation of the condylar head does not produce the click that is heard.
A 2-year-old boy presents with swelling over the bridge of the nose that has been present since birth. The swelling has been slowly increasing in size and he has hypertelorism. The swelling is soft, compressible, and it transilluminates. There are visible and palpable pulsations, and the mass enlarges when the patient cries. Which of the following is the most likely diagnosis? A ) Encephalocele B ) Glioma C) Hemangioma D) Nasal dermoid cyst E) Nasopharyngeal angiofibroma
The correct response is Option A.
Encephaloceles are neural tube defects that result in sac-like protrusions of the meninges (meningocele) or brain and meninges (meningoencephalocele) in various locations along the cranium, such as between the forehead and nose (including naso-orbital, frontonasal, and nasoethmoidal locations) or on the back of the skull. They tend to be soft, compressible masses that transilluminate that may be sessile or pedunculated. Biopsy may result in a cerebrospinal fluid leak.
Glioma is a mass of ectopic neural tissue that does not transilluminate.
Hemangiomas are benign vascular lesions that are present at birth and characterized by a rapid growth phase around the age of 1 to 6 months, followed by gradual involution over 1 to 12 years. They have no intracranial connection and no cerebral pulsations. Nasopharyngeal angiofibromas, also known as juvenile nasopharyngeal angiofibromas, are benign but locally
invasive vascular tumors that occur almost exclusively in adolescent males. They present with unilateral or bilateral nasal obstruction, frequent epistaxis or blood-tinged nasal discharge. Nasal dermoid cyst is a benign cystic lesion that does not pulsate and does not transilluminate.
Which of the following best represents the likelihood that a patient with a frontal sinus fracture would have a concurrent intracranial injury? A) 1% B) 15% C) 30% D) 55% E) 90%
The correct response is Option D.
In an acute trauma setting, the recognition of mild traumatic brain injury (mTBI) is a diagnostic challenge as there are often competing diagnoses that take immediate priority. Furthermore, within this cohort, patients with craniofacial fractures have been shown to be at risk for delayed or missed diagnosis for all degrees of TBI, although with a higher likelihood of missed or delayed diagnosis for mTBI compared with moderate to severe TBI. Previously, it was hypothesized that facial fractures buffered the forces transmitted during blunt head trauma, thereby protecting intracranial structures. This conceptual framework has since been questioned as evidence has mounted that individuals with facial fractures are at increased risk for head injury. The biomechanics resulting in different types of facial fractures and the amount of force required to fracture the different components of the facial bony structure have been well described. The nasal bone has the lowest tolerance for fracture at 25 to 75 lbs, while the frontal bone has the highest tolerance at 800 to 1600 lbs. Recent studies have proposed that craniofacial fractures can serve as clinical markers for brain injury and Mulligan et al. suggest that the prevalence of overall head and cervical spine injuries in the setting of facial fractures is high enough to warrant a change in current protocols.
In this context, the prevalence of mTBI and moderate to severe TBI in patients with isolated facial fractures in the National Trauma Databank (NTDB) was evaluated, and further characterized the association of isolated facial fractures with different degrees of TBI in patients with mild, moderate, and severe TBI. Facial fractures can serve as objective clinical markers for the potential presence of mTBI and moderate to severe TBI in trauma patients. As mTBI patients have been shown to benefit from simple, easy-to-administer educational interventions, trauma patients with facial fractures may benefit from automatically receiving education about mTBI and TBI recovery, given the clinically meaningful prevalence of mTBI and TBI in this population. As one moves up the craniofacial skeleton, the forces are
transmitted more reliably to the intracranial space. Therefore, a frontal sinus fracture is at extremely high risk (usually a 45 to 65% chance) of having an associated intracranial injury.
A 55-year-old man presents with a 5 × 5-cm open wound on the medial lower leg with healthy granulation tissue. A split-thickness skin graft from the upper thigh is planned. Which of the following dressings would result in the highest degree of pain at the donor site? A) Alginate B) Hydrocolloid C) Hydrofiber D) Petrolatum gauze E) Semipermeable
The correct response is Option D.
Petrolatum-based dressings, such as Adaptic or Xeroform, are considered nonmoist dressings. Moist dressings include semipermeable (Tegaderm or OpSite), hydrofiber (Aquacel), alginate, or hydrocolloid (DuoDERM).
In a recent meta-analysis from 2018, 41 articles with 35 prospective randomized trials found that moist dressings at a skin graft donor site are associated with less pain as compared to nonmoist dressings. When examining various time points in recovery, it was found that moist dressings showed improved pain control at every time point (postoperative days 1 to 4, 4 to 7, greater than 8).
With regard to wound healing, this finding was not universal, although most studies showed that moist dressings demonstrated improvements in re-epithelialization rate and quality of healing. Infection and cosmetic outcome were also not significantly different with variable results.
A 24-year-old woman undergoes facial rejuvenation with hyaluronic acid gel filler in the glabellar region. Several hours later, she develops skin discoloration and discomfort across the central forehead. She presents to the office for evaluation 2 days later. On physical examination, skin blanching with decreased temperature and paresthesias are confirmed. Which of the following is the most effective intervention to treat this patient?
A) Administer aspirin 325 mg
B) Administer injection of hyaluronidase
C) Apply topical nitroglycerin
D) Apply warm compress and massage
E) Offer reassurance and observation
The correct response is Option B.
This clinical scenario depicts acute arterial vascular compromise following injection of hyaluronic acid into the glabellar region. The symptoms of painless blanching and decreased sensation suggest early ischemic changes. Both arterial and venous structures can be injured from filler injections through direct injury, embolic occlusion, or a combination of the two. Although rare, this complication requires prompt recognition and treatment to reverse the ischemic process and prevent tissue necrosis. In a retrospective review, Park et al. identified two areas that are particularly susceptible to necrosis because of lack of collateral circulation: the glabella and the nasolabial fold near the alar subunit.
Reassurance and observation are not appropriate for evolving tissue necrosis. Application of warm compresses and topical nitroglycerin will cause vasodilation, which will help improve local blood flow, but the most appropriate treatment is injection of hyaluronidase, an enzyme that cleaves the peptide bonds of long chain proteins within hyaluronic acid, thereby increasing the permeability of the filler and facilitating dispersion and absorption. Aspirin has not been shown to provide any benefit.
REFERENCES:
A 45-year-old African American woman who underwent cardiac surgery 6 months ago seeks cosmesis of the surgical scar. Physical examination shows a raised, thick keloid scar that is pruritic. Which of the following nonsurgical treatments is likely to produce the most rapid improvement in the scar with the fewest adverse effects? A) Intralesional 5-fluorouracil B) Intralesional triamcinolone C) Radiation therapy D) Silicone sheeting E) Topical triamcinolone
The correct response is Option A.
Injection of 5-fluorouracil demonstrates similar efficacy to intralesional corticosteroid therapy (triamcinolone) and has the advantage of a lower risk of hypopigmentation.
Silicone sheeting is effective for hypertrophic scars but requires serial application and is slower in visible effect. Topical corticosteroids are not as effective as intralesional treatment. Radiation carries the risk of adjacent tissue toxicity and also includes the risk of skin pigmentation changes.
A 15-year-old girl has a 12-month history of pain and fullness in the right supraorbital rim. She has café-au-lait spots, a history of precocious puberty, and a recent pathologic rib fracture. Which of the following is the most likely pathology of the lesion? A) Dermoid cyst B) Fibrous dysplasia C) Neurofibroma D) Osteoblastoma E) Rhabdomyosarcoma
The correct response is Option B.
This patient has a classic presentation of McCune-Albright syndrome. Patients with McCuneAlbright present with a triad of polyostotic fibrous dysplasia, precocious puberty, and skin pigmentation (eg, café au lait spots). Additionally, they may have hyperfunctioning
endocrinopathies such as growth hormone excess. If these patients present with intramuscular myxomas, it is known as Mazabraud syndrome. Malignant degeneration of fibrous dysplasia has been reported in up to 4% of patients with McCune-Albright syndrome. Management depends on the clinical presentation and functional impact of the lesions, and is primarily surgical.
Patients with neurofibroma may have café au lait spots, but not precocious puberty or pathologic fractures. Dermoid cysts generally do not present with pain and are often noted at a much younger age than the patient described.
A 57-year-old woman undergoes microsurgical breast reconstruction using a muscle-sparing transverse rectus abdominis musculocutaneous (MS-TRAM) flap. Near-infrared spectroscopy (NIRS) is used to monitor the flap in the postoperative setting. NIRS measures which of the following parameters?
A) Arterial oxygen saturation (SaO2)
B) Mixed venous oxygen saturation (SvO2)
C) Partial pressure of oxygen (PaO2)
D) Peripheral oxygen saturation (SpO2)
E) Tissue oxygen saturation (StO2)
The correct response is Option E.
Near-infrared spectroscopy (NIRS) is a noninvasive modality that allows continuous monitoring of tissue oxygenation and perfusion. It measures relative changes in the concentration of oxygenated and deoxygenated hemoglobin. Tissue oxygen saturation (StO2) is the percentage of hemoglobin in tissue that is oxygenated. Since StO2 measures oxygen saturation in the vascular bed of tissue, it measures both venous and arterial saturation and, thus, reflects both oxygen delivery and consumption. This provides a good surrogate for tissue perfusion. Peripheral capillary oxygen saturation (SpO2), measured by pulse oximetry, measures arterial oxygen saturation, which may not reflect perfusion. Arterial (SaO2) and mixed venous oxygen saturation (SvO2) as well as the partial pressure of oxygen (PaO2) are measured directly from blood and are indicative of systemic rather than local tissue oxygenation.
A 63-year-old man presents to the office with a 4 × 4-cm heel defect. A local podiatrist has debrided the calcaneus, and there is healthy granulation tissue present. After extensive discussion with the patient, it is decided to proceed with a distally based sural fasciocutaneous flap for coverage. Which of the following risk factors is associated with the highest rate of flap-related complications? A ) CAD B ) DM C) Hypertension D) Obesity E ) Venous insufficiency
The correct response is Option E.
A recent meta-analysis of 61 papers showed that venous insufficiency is associated with a nine-fold increase in risk of developing a complication in a distally based sural flap. Other risk factors, such as peripheral vascular disease, diabetes, obesity, and hypertension, have been associated with increased rates of complications previously in the literature, but venous insufficiency is associated with the highest rate of complications. The pooled data from this series showed a 26% complication rate, a 3.2% flap loss rate, and a 15.3% partial flap loss rate. As the design of the sural flap is distally based, the physiology of the flap requires reversed flow through the venous system.
Advanced age is also associated with an increased complication rate and there is literature to suggest a delay procedure in this patient population. Other papers have found smoking to have a higher risk of complications as well. There is still much debate over which patients should undergo a delay procedure or venous supercharging, but in general, high-risk patients with multiple comorbidities should be considered for this additional surgery. Coronary artery disease has not been found to be associated with increased risk of flap- related complications.
To reconstruct a traumatic soft-tissue defect of the hand, a lateral arm fasciocutaneous flap is chosen for free tissue transfer. The vascular pedicle for this flap is located between which of the following structures?
A) Brachioradialis and brachialis
B) Lateral head of the triceps and biceps
C) Lateral head of the triceps and the brachialis
D) Lateral head of the triceps and the extensor carpi radialis longus
E) Lateral head of the triceps and the humerus
The correct response is Option C.
The lateral arm flap is a reliable fasciocutaneous flap used for reconstruction of small to medium size soft-tissue defects of the forearm and hand. It provides thin tissue of excellent quality and a satisfactory aesthetic outcome with the ability for primary closure of the donor site.
The dominant pedicle is the posterior radial collateral artery. This vessel originates from the radial collateral artery which is a branch of the brachial artery in the upper arm and emerges between the brachialis muscle anteriorly and the lateral head of the triceps posteriorly to supply the skin and soft tissue of the lateral arm.
In addition, the anatomic relationships between the vascular pedicle and radial nerve are important to understand when dissecting this flap. The radial nerve courses posterior to the humerus before wrapping around the lateral aspect of the humerus from posterior to anterior. The antebrachial cutaneous nerve of the forearm branches from the radial nerve to join the vascular pedicle (posterior branch of radial collateral artery). This cutaneous nerve can be preserved with careful dissection and helps to identify the radial nerve proximally during flap elevation. Segmental perforating vessels emerge within the septum separating the triceps and brachialis muscles.
An 18-year-old man desires correction of a unilateral congenital Stahl ear deformity. Which of the following auricular characteristics is most likely to be observed on physical examination?
A) Accessory third crus of the antihelix
B) Conchal projection secondary to prominent mastoid
C) Enlarged conchal cartilage
D) Hyperplasia of the superior crus of the antihelix
E) Pointed thickening at the junction of the upper and middle third of the helix
The correct response is Option A.
Stahl congenital ear deformity can present with varying degrees of severity and is characterized by the presence of an abnormal third crus of the antihelix. There is also often an associated flattening of the helix, unfurling of the helical rim, a posterosuperior projection of the helical rim, and absence or hypoplasia of the superior crus of the antihelix.
A pointed thickening at the junction of the upper and middle third of the helix is seen in Darwin’s tubercle. Conchal projection secondary to prominent mastoid can be seen in mastoid prominence. Enlarged conchal cartilage is seen in prominent ear.
A healthy 5-year-old boy is scheduled to undergo a staged expansion to remove a large congenital melanocytic nevus. Which of the following anatomical regions of the body has the highest risk for complications during the expansion process? A) Head B) Lower extremity C) Neck D) Torso E) Upper extremity
The correct response is Option B.
The anatomic region associated with the most complications is the lower extremity. Huang et al. conducted a 20-year meta-analysis and systematic review to come to this conclusion:
• The pooled odds ratio for lower limb complications as compared with other anatomical sites was 2.80 (95% confidence interval, 1.14 to 6.86; p = 0.17).
• The pooled odds ratio for head and neck complications versus other anatomical sites was 1.31 (95% confidence interval, 0.95 to 1.80; p = 0.1).
• The pooled odds ratio for upper limb complications was 1.24 (95% confidence interval, 0.60 to 2.55).
• The pooled odds ratio for trunk complications was 0.78 (95% confidence interval, 0.45 to 1.33).
The results indicated that the lower limb was more likely to develop complications than other anatomical sites, whereas the trunk was the safest site on which to perform tissue expansion. This conclusion has also been reported by prior studies. There are certain anatomical and physiologic differences between the extremities and other areas. The extremities tend to be in frequent motion, resulting in compressive and disruptive forces exerted by the regional musculature. Scarring can limit the availability of local flaps because the flaps themselves require a complex geometry in terms of distance and orientation to effectively encompass a curvilinear biconvex surface. The creation of a pocket from a distance, around a curve, or in a cylindrical extremity can be quite challenging.
A 50-year-old woman presents to the clinic to discuss breast reconstruction after bilateral mastectomy. She is interested in free tissue transfer. She has a diagnosis of systemic lupus erythematosus treated with chronic steroid therapy and wants to know if she is an appropriate candidate for free flap reconstruction. Which of the following statements best describes the surgical risks for this patient with lupus compared with the general population?
A) Higher rate of free flap failure
B) Higher risk of a thromboembolic event
C) Similar rate of hernias after abdominally based free flaps
D) Similar rates of infection
The correct response is Option B.
The statement which best describes the surgical risks for a patient with lupus undergoing free tissue transfer for breast reconstruction is that the patient has a higher risk of a thromboembolic event than the average patient.
The rate of free flap failure in patients with lupus is similar to patients without lupus. Chronic steroid use increases the risk of wound healing complications in patients with lupus, rather than increases the risk of free flap failure. Additionally, patients with lupus have an increased risk of abdominal wall bulge and hernia after abdominally based free flaps compared with the average population. Chronic steroid use also suppresses the immune system, predisposing patients treated with steroids to increased rates of infection compared to patients not taking steroids.
A newborn presents with an asymmetric bilateral cleft lip-nose-palate; the right side is incomplete and the left side is complete. A photograph is shown. Which of the following sequences of repair is most likely to provide the optimal outcome?
A) One-stage repair of the bilateral cleft lip
B) Repair cleft palate at the initial surgery
C) Repair the complete side cleft lip first, then stage incomplete side cleft lip at a later surgery
D) Repair the incomplete side cleft lip first, then stage complete side cleft lip at a later surgery
E) Simultaneous repair of cleft lip and cleft palate
The correct response is Option A.
Bilateral cleft lip repair is more commonly symmetric and is usually repaired as a one-stage repair around 3 to 6 months of age.
Asymmetrical bilateral cleft lip repair can be done in one stage or two stages, and the sequencing has been controversial. However, two recent studies show convincing evidence that even for asymmetrical bilateral cleft lip repairs, a one-stage repair leads to overall better symmetry and cleft lip repair outcomes. Therefore, a two-stage bilateral cleft lip repair is incorrect.
Cleft palate repair is typically performed closer to 9 to 12 months of age, therefore, cleft palate repairs at 6 to 9 months of age are less typical and irrelevant, as the two-stage repair is considered not optimal for this asymmetric bilateral cleft lip scenario.
A 68-year-old woman who underwent a three-vessel coronary artery bypass grafting (CABG) 7 weeks ago presents to the office with a 6-mm indurated lesion at the manubrium of the healed sternotomy incision. She also has two draining lesions 1 and 1.5 cm lateral to the sternotomy incision. The patient says that these two lesions began draining approximately 3 weeks ago. The CABG included harvest of the left internal mammary artery. CT scan of the chest demonstrates a sinus tract leading to a 4 × 6-cm fluid collection in the substernal region. After removal of sternal wires and debridement of necrotic bone and cartilage, which of the following is the most appropriate reconstructive option?
A) Bilateral pectoralis major advancement flaps
B) Bilateral pectoralis major turnover flaps
C) Left rectus abdominis muscle flap
D) Omental flap
E) Right latissimus dorsi flap
Upon further review, this item was not scored as part of the examination.
The correct response is Option A.
The most common and potentially devastating complication of median sternotomy is infection resulting in wound breakdown. Other complications seen less frequently are mechanical breakdown of the wound secondary to inadequate wiring technique, poor bone quality, and chronic obstructive pulmonary disease (COPD) associated with uncontrolled, severe coughing. Before the use of muscle flap closure of the sternal defect, mortality was in the 50% range. Subsequent to the development of muscle flap techniques, this has been dramatically reduced to the 15 to 30% range and even lower in many institutions (< 5%).
The treatment options for sternal wound closure include unilateral pectoralis major muscle turnover flap, the unipedicle pectoralis major muscle rotation advancement flap, the rectus abdominal muscle flap, bilateral myocutaneous pectoralis major muscle flaps, latissimus dorsi muscle flap, and the omental flap. Pectoralis major advancement flaps based on the thoracoacromial vessels are the first line option for coverage of sternal wounds. The left internal mammary artery has been harvested in this case, compromising the vascularity to the left pectoralis major turnover flap and the left rectus abdominis flap. The latissimus dorsi flap is a second line choice the presence of other reconstructive options. Omental flap is used when a large mediastinal dead space needs to be filled.
Sternal wounds are divided into three categories: acute (those occurring within the first
2 postoperative weeks of sternotomy), subacute (those occurring between the second and fourth postoperative weeks), and chronic (those occurring any time thereafter).
Although chronic wounds may present with all of the life-threatening symptoms of the acute or subacute wounds, they usually present as wounds with a small amount of drainage, commonly from several sites. Osteomyelitis, a feature of many of the sternal wounds, is common to this group. Often this occurs at sites of wire placement. These wounds are treated in a similar fashion to the subacute wounds, requiring wire removal with thorough debridement including involved bone and cartilage prior to muscle or omental flap coverage.
The preoperative incision and drainage of the sternal wound, even as little as 48 hours before definitive repair, reduced the reinfection rate from 15% early in the Lenox Hill series to 5% later in the series. The authors’ strategy was to prepare the opened wound for final closure with intense local wound care with wet to dry dressings every 6 hours or with wound vac therapy. They found reinfection could be minimized if surgery was avoided in the presence of frank purulence.
A 38-year-old African American man presents with multiple purulent tunneling lesions in bilateral axillae and his right groin. Which of the following surgical treatments will result in the lowest likelihood of recurrence? A) Deroofing B) Electrosurgical peeling C) Incision and drainage D) Skin-tissue-saving excision E) Wide excision
The correct response is Option E. Hidradenitis suppurativa (HS) is an inflammatory skin disease with a characteristic clinical presentation of recurrent or chronic painful or suppurating lesions in the apocrine gland- bearing regions. HS should be differentiated from infections such as furuncles, carbuncles, and abscesses (due to infectious agents and response to antibiotics), cutaneous Crohn disease (often concurrent with gastrointestinal Crohn, which has “knife-cut” ulcers and no comedones [whiteheads or blackheads]), and acne (distributed on the face and upper truncus, whereas HS predominantly affects intertriginous areas). Surgery is required to definitively treat the tunnels and scars associated with chronic HS. Although surgery is commonly recommended, the literature supporting surgical treatment is anecdotal, composed mostly of large case series or retrospective study reports. A systematic review by Mehdizadeh et al. concluded that a lower recurrence rate was found in procedures with wide excision (overall, 13%; primary closure, 15%; using flaps, 8%; grafting, 6%) compared with local excision (22%) or deroofing (27%). These operations can be disfiguring, and despite the removal of significant amounts of tissue, do not necessarily protect against disease recurrence.
A healthy 55-year-old woman underwent bilateral breast reconstruction with free deep inferior epigastric perforator (DIEP) flaps. Tissue oximetry-based flap monitoring is used. Which of the following is the main advantage of this technique over a hand-held Doppler with clinical assessment? A) Direct blood flow measurement B) Ease of use C) Improved flap salvage rate D) Less expensive modality E) Operator must be bedside
The correct response is Option C.
The main advantage of using tissue oximetry-based monitoring is that it improves flap salvage rates. Tissue oximetry, or near-infrared spectroscopy, is increasing in popularity among microsurgeons and has been shown to be the third most commonly used technique after clinical examination and hand-held Doppler. Rather than directly monitoring flow, tissue oximetry uses infrared light to measure the relative concentrations of oxygenated and deoxygenated hemoglobin. By measuring oxygenation rather than flow, the probe is relatively unaffected by movement artifacts. Recent studies emphasize its value in identifying flap compromise before clinical signs of arterial or venous thrombosis. In a 2011 study, Lin et al. reported an increased flap salvage rate at their institution with the use of near-infrared spectroscopy, from 57.7 to 93.8% (p = 0.015), despite no significant increase in their rate of reexploration, attributing this improvement to earlier recognition of vascular compromise. In a recent small prospective cohort study, Lohman et al. followed 38 free flaps with physical examination and five technologies, including handheld Doppler, implantable Doppler, and tissue oximetry. Although primarily a descriptive study, they concluded that tissue oximetry was the first technology to record signs of flap compromise.
Though tissue oximetry-based flap monitoring is easy to use, so is a hand-held Doppler, so that is not the main advantage. It does have a higher financial investment to buy the system, but over time it could be argued it more than pays for itself given the improved flap salvage rates. Unlike the hand-held Doppler, this modality has a continuous read on the monitor, the examiner need not be in the presence of the patient, and, in fact, can visualize the readings on a smart phone through an app.
A 15-year-old boy sustained a traumatic amputation of the left index finger at the proximal interphalangeal joint level from a sharp injury. Replantation of the digit is performed, with vein grafting of the radial digital artery and vein. The distal tip of the digit appears congested, so medicinal leeching is instituted. Which of the following antibiotics is the most appropriate prophylaxis for this patient? A) Amoxicillin and clavulanic acid B) Ampicillin C) Cephalexin D) Ciprofloxacin E) Vancomycin
The correct response is Option D.
The antibiotic choice that constitutes the best prophylaxis for this patient undergoing leech therapy is ciprofloxacin. Hirudo medicinalis is the most common leech species used in medicine, and its gut flora includes Morganella, Rikenella, and Aeromonas isolates. These bacteria are all sensitive to ciprofloxacin. Doxycycline or ceftriaxone are alternative treatments for Aeromonas prophylaxis.
Animal toxicology data available with the first quinolone compounds revealed an association with inflammation and subsequent destruction of weight-bearing joints in canine puppies. This observation limited further development or large-scale evaluation of this class of antibiotic agents in children at that time. However, there continued to be increased use of fluoroquinolones for pediatric patients over the past 30 years with data on the lack of toxicity when used in children. In 2004, ciprofloxacin became the first fluoroquinolone agent approved for use in children 1 through 17 years of age.
Cephalexin (Keflex) is a first-generation cephalosporin that is used to treat respiratory tract, middle ear, skin, bone, and urinary tract infections. Most Aeromonas strains are resistant to penicillin, ampicillin, carbenicillin, and ticarcillin. And most Aeromonas and Morganella strains have complete or intermediate resistance to amoxicillin and clavulanic acid (Augmentin). Vancomycin is a macrolide antibiotic, and has limited effectiveness
for Aeromonas strains with high levels of antibiotic resistance.
A 50-year-old woman with type 2 diabetes mellitus is scheduled to undergo ligament reconstruction tendon interposition (LRTI) surgery for trapeziometacarpal joint arthritis. The procedure is expected to last 90 minutes. Which of the following is the most appropriate antibiotic prophylaxis for this patient?
A) Oral antibiotics for 3 days following surgery
B) Single dose intravenous antibiotic within 1 hour of surgery
C) Single dose intravenous antibiotic within 1 hour of surgery and oral antibiotics for 24 hours following surgery
D) Single dose intravenous antibiotic within 1 hour of surgery and oral antibiotics for 3 days following surgery
E) No antibiotic prophylaxis is indicated
The correct response is Option E.
Multidrug resistant bacterial infections continue to rise and antimicrobial overuse is the leading cause for antibiotic resistance. There is growing evidence that prophylactic antibiotic use is not necessary for clean plastic surgery cases, aside from breast surgery cases. Despite consensus guidelines, the use of prophylactic antibiotics for elective Hand Surgery cases continues to increase. Level I evidence exists that demonstrates prophylactic antibiotics are not necessary for clean Hand Surgery cases lasting less than 2 hours. Although there has been concern regarding diabetes and surgical infection risk, this has not been demonstrated in larger studies with multivariate analyses.
A 74-year-old woman presents with biopsy-proven Merkel cell carcinoma involving the left cheek and no palpable lymphadenopathy. The most appropriate treatment involves wide local excision with 2-cm margins and which of the following?
A) Adjuvant radiation and immunotherapy
B) Adjuvant radiation only
C) Cervical lymphadenectomy
D) Sentinel lymph node biopsy and adjuvant radiation
E) No additional intervention is needed
The correct response is Option D.
Merkel cell carcinoma is a rare and aggressive neuroendocrine cutaneous malignancy with a propensity for lymph node and distant metastasis and high mortality. It typically presents as an asymptomatic erythematous papule or nodule in the head and neck region. Surgical treatment includes wide local excision of the primary lesion with 1- to 2-cm margins to achieve histologically clear margins. Sentinel lymph node biopsy has become standard of care and helps determine prognosis and may guide additional adjuvant therapy. Negative sentinel node biopsy correlates with longer disease-free survival, especially in early stage disease. Adjuvant radiation has been shown in improve local-regional control and increase survival. Cervical lymphadenectomy would only be indicated in the setting of clinically positive lymph nodes and is insufficient as the sole adjuvant therapy.
A 48-year-old man presents with an asymptomatic, slowly growing violaceous nodule overlying his right scapula. Dermoscopy shows a delicate pigmented network, vessels, and structureless light brown areas. Biopsy identifies the lesion as a dermatofibrosarcoma protuberans. To minimize recurrence, surgical excision should include which of the following? A) Bone B) 6-cm margin C) Deep fascia D) Frozen sections E) Periosteum
The correct response is Option C. Dermatofibrosarcoma protuberans (DFSP) is a rare neoplasm of intermediate malignancy. Taylor first described it in 1890, but Darier is credited with establishing DFSP as a distinct clinicopathological entity in 1924, and finally, Hoffman established the term in 1925. Most DFSPs occur on the trunk (42%), followed by the upper extremities (23%), lower extremities (18%), then the head and neck (16%). The reason for recurrence is that microscopic projections are not removed adequately or assessed satisfactorily; the wider the margin, the higher the probability the tumor will be removed completely. Using 1-cm margins around the primary tumor leaves residual microscopic tumor in more than 70% of patients, 2- cm margins in 20 to 40% of patients, 3-cm margins in 9 to 15.5% of patients, and 5-cm margins in 5% of patients. Peripheral margins of 5 cm have a close to 0% recurrence rate. Wide resections may also not be practical in patients with tumors located in critical areas like the head and neck, and intraoperative frozen section assessments have not been reliable for determining margin status. Deep margins should always include excision of the deep fascia. The complete resection of the tumor requires excision of the external outer table in the cranium; muscle in trunk and extremities; peritoneum in thin patients with DFSP located on the abdomen, sternum, and clavicles; and vertebral apophysis when located on the thorax.
A healthy 20-year-old woman presents with a deep abrasion injury of the left volar forearm that she sustained in a bicycle accident. Physical examination of the forearm shows exposed flexor tendons. The wound is debrided, and no vital nerves or vessels are exposed. The wound is approximately 4 × 4 cm. A bilaminate acellular dermal regenerative matrix is used to reconstruct the defect. The application of negative-pressure therapy is most likely to result in which of the following outcomes in this patient?
A) Increase in matrix take
B) Increase in timing to definitive skin graft
C) Negative-pressure therapy is not indicated
D) No change in matrix take
E) No change in timing to definitive skin graft
The correct response is Option A.
The application of negative-pressure therapy results in an increase in matrix take. Dermal regeneration matrix (DRM) (clinical example is Integra) is a mesh-bilayered acellular matrix composed of a cross-linked bovine tendon collagen and glycosaminoglycan dermal equivalent, and a semipermeable polysiloxane (silicone) epidermal equivalent. It is a biodegradable matrix that acts as a scaffold for fibroblast and endothelial invasion and capillary growth. DRM has been approved by the FDA (Federal Drug Administration) for use in partial- and full-thickness wounds, pressure ulcers, venous leg ulcers, surgical wounds, chronic vascular ulcers, second-degree burns, and draining wounds.
The combination of DRM with fibrin glue and postoperative negative-pressure therapy versus DRM alone in acute and chronic wounds increased the take rate from 78 to 98%. Moreover, the interval between DRM and skin transplantation was decreased from 24 to 10 days, thus decreasing the length of hospital stay (Level II evidence).
The other options are therefore incorrect. Negative-pressure therapy is commonly used in these clinical scenarios.
A 55-year-old diabetic woman with a history of breast cancer is scheduled to undergo autologous breast reconstruction with a pedicled transverse rectus abdominis musculocutaneous (TRAM) flap. The surgeon is planning to ligate the inferior epigastric vessels in advance of the breast reconstruction. Which of the following mechanisms best describes how the delay phenomenon augments blood flow?
A) Decreased levels of prostaglandin E2
B) Increased levels of prostaglandin F2
C) Ischemia-induced dilation of blood vessels
D) Ischemia-induced hyperplasia and hypertrophy of blood vessels
E) Reorientation of choke vessels along the short axis of the flap
Upon further review, this item was not scored as part of the examination.
The correct response is Option D.
The delay phenomenon is also known as ischemic preconditioning. Essentially a flap is rendered partially ischemic, which results in neovascularization over a period of time. This can promote flap survival, increase the length-to-width ratio in random pattern flaps, and ensure the reliable transfer of larger volumes in axial pattern flaps.
Vascular delay causes hyperplasia and hypertrophy of the vessel walls during the first 48 to 72 hours of ischemia. Transected sympathetic nerves release norepinephrine into the tissue during early ischemia. This leads to a hyperadrenergic state, which causes vasoconstriction for up to 30 hours and augments early ischemia. The choke vessels reorient along the long axis of the flap, which enhances blood supply to the distal regions of the flap tissue. These tend to be the areas most prone to necrosis. There is an increase in prostaglandin E2 (a vasodilator) and a decrease in prostaglandin F2 or thromboxane (a vasoconstrictor) during late ischemia. This minimizes ischemia and enhances flap survival.
Which of the following comorbidities is associated with the highest risk of digital replant failure? A) Alcohol abuse B) Chronic obstructive pulmonary disease C) Diabetes mellitus D) Psychotic disorders E) Tobacco use
The correct response is Option D.
In a study looking at all amputation injuries and digital replantations captured by the National Inpatient Sample from 2001 to 2012, the comorbid conditions associated with the highest risk of replant failure were psychotic disorders, peripheral vascular disease, and electrolyte imbalances. The risk of replant failure increased 79% in a patient with a psychotic disorder. Alcohol abuse increased the risk of replant failure by 16%, tobacco use by 7%, diabetes by 3%, and chronic obstructive pulmonary disease by 1%. Interestingly, age in and of itself was not associated with a higher chance of replant failure in this and other studies.
A 30-year-old woman comes to the office to discuss surgical augmentation of the chin. Which of the following outcomes is most likely in this patient if a porous polyethylene prosthesis is used instead of a solid silicone rubber prosthesis?
A) Increased incidence of bone resorption
B) Increased incidence of infection
C) Increased ingrowth of tissue
D) Increased likelihood of malposition
E ) Reduced resorption of the implant
The correct response is Option C.
Porous polyethylene implants have enough rigidity to resist soft-tissue deforming forces but enough flexibility to facilitate placement. The pore size (diameter of 100 to 250 μm) of porous polyethylene used in facial augmentation procedures is sufficient to allow fibrous tissue ingrowth and relative incorporation of the prostheses. This avoids the capsule formation intrinsic to smooth-surface implants which is the result of the host’s foreign body response. This superficial tissue integration makes porous polyethylene facial prostheses less likely to migrate after implantation than solid silicone prostheses, but it also makes their explantation more difficult compared with solid silicone prostheses.
Silicone rubber has a smooth surface and is relatively flexible, making implant placement and removal beneath the soft-tissue envelope easier.
Neither porous polyethylene nor silicone prostheses are resorbed after implantation. Two recent studies reported on a total of 53 patients undergoing chin augmentation with silicone implants. The authors found 55% of those patients experienced underlying bone resorption during the 20 month follow-up period based on lateral radiographs.
A 76-year-old woman presents with a diagnosis of recurrent adenocarcinoma of the rectum after chemotherapy and radiation. The planned oncologic surgical treatment will be an abdominoperineal resection to include the rectum, anus and posterior wall of the vagina, and left end colostomy. She has had a prior laparoscopic bowel resection, but no open abdominal surgeries. Which of the following is the most appropriate surgical management for her planned perineal wound?
A) Bilateral pudendal flaps
B) Omental flap and skin graft
C) Primary perineal closure
D) Right gracilis myocutaneous flap
E) Right vertical rectus abdominis myocutaneous flap
The correct response is Option E.
Abdominoperineal resection and pelvic exenteration in previously irradiated patients create a large- volume non-collapsible dead space in the pelvis that can lead to large persistent perineal wounds. Meta-analysis reviews of existing patient series report that primary perineal closure is associated with twice the rate of perineal wound formation as flap closure. When compared with thigh-based flaps, such as the gracilis flap and gluteal-based flaps, the vertical rectus abdominis
myocutaneous flap is associated with lower perineal wound and flap complication rates.
With prior abdominal surgery, the omentum may not be available or of sufficient volume for reconstruction. A skin graft is often applied to the omentum for sternal wound reconstruction, but would not be appropriate for resurfacing of the posterior wall of the vagina in concert with an omental flap.
Bilateral pudendal flaps would be in the previously irradiated field, increasing the potential for wound healing trouble with this flap selection. Additionally, pudendal flaps would not bring the bulk necessary to address an abdominoperineal resection defect.
Cutaneous squamous cell carcinoma arises from which of the following epidermal layers? A) Stratum basale B) Stratum corneum C) Stratum granulosum D) Stratum lucidum E) Stratum spinosum
Upon further review, this item was rekeyed to option E. Additionally, the rationale was revised as shown below.
The correct response is Option E.
Squamous cell carcinoma arises from keratinocytes in the stratum spinosum. The epidermis comprises four layers, from deep to superficial: stratum basale, stratum spinosum, stratum granulosum, and stratum corneum. Glabrous skin has an additional layer between the stratum granulosum and stratum corneum called stratum lucidum. The stratum basale consists of basal cells. These are stem cells that differentiate into keratinocytes. These keratinocytes then migrate upwards. In the stratum spinosum the keratinocytes form intercellular connections via desmosomes. In the stratum granulosum, the keratinocytes have keratohyalin granules. In the stratum corneum, the cells are compact and surrounded by a lipid layer, thus giving the skin its barrier function.
Melanocytes and Merkel cells are found in the stratum basale. Langerhans cells are found in the stratum spinosum, stratum granulosum, and the dermis. Basal cell skin cancers arise from the stratum basale.
A 66-year-old, 132-lb (60-kg) woman presents to the emergency department with a deep second-degree, 20% total body surface area burn, with a small area of surrounding first-degree burn. This happened in an open space when she fell backwards onto a fire pit. Her burns are isolated to her buttock and back. She reports no hoarseness or difficulty breathing. She receives 9600 mL of Ringer’s lactate within the first 24 hours. Based on her fluid resuscitation, she is at highest risk for which of the following?
A) Abdominal compartment syndrome
B) Conversion of the burn to full-thickness
C) Deep venous thrombosis
D) Digit ischemia
E) Poor engraftment of autologous skin grafts
The correct response is Option A.
Numerous formulas regarding burn resuscitation have been developed to avoid under- resuscitation. This was based on previous literature suggesting that under-resuscitation was associated with significant end organ damage secondary to ischemic injury. Inadequate resuscitation was similarly associated with the potential for hemodynamic collapse, resulting in death. Weight-based resuscitation programs and establishment of urine output guidelines have largely limited under-resuscitation at burn centers. However, burn patients now suffer from the consequences of over-resuscitation, in which patients receive even more fluid than recommended by the Parkland formula. These patients suffer from increased rate of burn infections, the development of acute respiratory distress syndrome, and abdominal compartment syndrome.
In the clinical case presented, the Parkland formula would suggest that the patient
receive approximately 4800 mL of resuscitation (4 × TBSA burn [20] × weight in kilograms [60]) within the first 24 hours. This patient received approximately double the amount.
Regarding engraftment, there is no evidence that over-resuscitation worsens autologous skin graft engraftment. Conversion of burn injuries to deeper injuries is usually associated with under-resuscitation, as is end organ or digit ischemia. Deep venous thrombosis does not have any reported correlation with burn resuscitation.
A 35-year-old affirmed male (birth-assigned female) presents to the office requesting top surgery for a more masculine chest appearance. On examination, the patient has a large C-cup breast, grade 2 ptosis, and a nipple-to-inframammary fold distance of 11 cm. Which of the following surgical options is the most appropriate? A) Circumvertical reduction mammaplasty B) Liposuction alone C) Mastectomy with free nipple graft D) Periareolar reduction mammaplasty E) Wise pattern reduction mammaplasty
The correct response is Option C.
The most appropriate surgical procedure in this patient is a subcutaneous mastectomy with free nipple graft due to breast size and ptosis. Top surgery, or excision of the female breast tissue and shaping of the male chest, is often the first surgical procedure for female-to-male transgender patients. This marks the beginning of their surgical transition into a masculine phenotype and is associated with profound and impactful psychological and aesthetic benefits for the individual. The goal of top surgery is to produce a normal-appearing male chest. This is achieved by removing the breast parenchyma, obliterating the inframammary fold, reducing the areolar size, and positioning the nipple-areola complex into the appropriate position.
There are numerous studies demonstrating excellent results and high patient satisfaction with subcutaneous mastectomy and free nipple grafts in patients with large and ptotic
breasts. Additionally, patients with a BMI over 27 kg/m2 and a nipple-to-inframammary fold distance greater than 7 cm demonstrate better outcomes with fewer complications. Periareolar breast reduction also demonstrates excellent results when limited to patients with smaller breasts, minimal ptosis, and a smaller skin envelope.
Liposuction alone will not achieve the goals above and will result in residual breast tissue and excess skin. Additionally, liposuction will not address areolar size or position. Liposuction is useful when combined with other techniques to help feather tissue thickness and contour the edges of the chest. Both Wise pattern and circumvertical reduction mammaplasty will leave behind too much breast tissue and will not create the desired masculine chest appearance.
Which of the following is associated with the use of pressure garments in the management of burn scars?
A) Decreased scar strength
B) Increased synthesis of tissue proteinases
C) Larger and less densely packed collagen fibers
D) Reduced differentiation of fibroblasts to myofibroblasts
The correct response is Option D.
Significant differences in scar contraction were observed between scars receiving pressure garment therapy and control burns that received no pressure. Pressure garments exert compressive forces perpendicular and parallel to the surface of the scar. These forces oppose the direction of contracture. One hypothesis is that wound tension acts upon integrins by stretching them, which leads to phosphorylation of focal adhesion kinase and upregulation of smooth muscle actin and collagen production. When compression is applied to incisional wounds perpendicular to the wound tension, scarring is minimized. This suggests that the mechanical forces applied to the scar can assist in reducing differentiation of fibroblasts to myofibroblasts, decreasing scar contraction and collagen deposition.
Scar strength was improved with pressure garment therapy compared with controls, with a 34% increase in ultimate tensile strength. Pressure garment therapy scars were also found to be composed of smaller, more densely packed collagen fibers.
Increased synthesis of tissue proteinases is a mechanism of corticosteroids.
A 43-year-old man presents with a recurrent 6-mm skin lesion on his upper lip. It is characterized by an erythematous papule surrounded by scale and crust. Biopsy shows nests of poorly differentiated cords of spindle cells of keratinocyte origin. Which of the following is the most appropriate treatment? A) Cryosurgery B) Electrodessication and curettage C) Mohs micrographic surgery D) Radiation therapy E) Standard surgical excision
The correct response is Option C.
The patient’s histology is consistent with squamous cell carcinoma. Mohs micrographic surgery (MMS) has the highest cure rates for both primary and recurrent squamous cell carcinoma. For locally recurrent tumors, the 5-year cure rate for MMS is 90% compared with 76.7% for standard surgical excision. Because MMS is a tissue-sparing technique, smaller surgical margins are taken, and scarring and functional impairment are minimized compared with standard surgical excision and electrodessication and curettage. Tumor removal and reconstruction are usually performed on the same day, using local anesthesia in an office- based setting. Recurrence rates after cryosurgery and radiation therapy are not as favorable as for MMS.
A 57-year-old woman presents with inability to extend the right thumb at the interphalangeal joint for the past 3 months. The patient reports a dull aching of the right wrist for several weeks before the sudden loss of thumb function. On physical examination, she cannot lift her right thumb when her palm is placed on a flat surface. Seven months ago, she was treated in a cast for a non-displaced right distal radius fracture; the fracture healed uneventfully. Which of the following is the most appropriate treatment to restore thumb function in this patient?
A) Arthrodesis of the interphalangeal joint
B) Lengthening of the flexor pollicis longus
C) Posterior interosseous neurolysis
D) Primary repair
E) Tendon transfer
The correct response is Option E.
This patient has sustained a rupture of the extensor pollicis longus (EPL) tendon. Tendon transfer, using the extensor indicis proprius (EIP) tendon, is the most appropriate way to restore function to the thumb. Although head-to-head comparison studies do not exist, the other options listed would not be expected to restore the patient’s function, based on the clinical scenario presented.
EPL rupture is not uncommon after a distal radius fracture. Although it may not be intuitive, EPL ruptures are actually more common following non-displaced fractures than displaced fractures. The cause of rupture is thought to be ischemic damage leading to attritional rupture of the tendon within the tight fibro-osseous tunnel where the EPL routes around the dorsal tubercle of the radius (Lister tubercle). Because this is an attritional rupture, primary tendon repair is usually not feasible, especially not 3 months after the rupture has occurred.
Tendon grafting is possible but has the disadvantage of requiring two separate
tenorrhaphies and a graft donor site. Both tendon transfer and/or tendon grafting may be done under local anesthesia to possibly help set tension more accurately. An
interphalangeal (IP) joint arthrodesis might help stabilize an unstable IP joint if no tendon reconstruction is possible. However, this procedure is not indicated in most EPL ruptures as it does not restore the missing function(s).
Flexor tendon lengthening is not indicated; this patient does not have a tight or contracted flexor tendon. Posterior interosseous nerve (PIN) neurolysis may be indicated in cases of PIN compression (gradual or insidious onset, pain in forearm, weakness of multiple muscles),
but the patient described in this scenario does not have these complaints.
A 26-year-old man sustains a laceration to the left palm with a hunting knife. Injury to the flexor digitorum superficialis (FDS) of the ring finger is suspected. Which of the following maneuvers performed on this patient is the most reliable method to examine FDS tendon function?
A) Flexion of the distal interphalangeal (DIP) joint with the other fingers held in extension
B) Flexion of the DIP joint with the ring finger metacarpophalangeal (MCP) and proximal interphalangeal (PIP) joints in extension
C) Flexion of the DIP joint with the ring finger MCP joint and PIP joints in flexion
D) Flexion of the PIP joint with the other fingers held in extension
E) Flexion of the PIP joint with the ring finger MCP joint in extension
The correct response is Option D.
The most reliable method for examining flexor digitorum superficialis (FDS) tendon function is with flexion of the proximal interphalangeal (PIP) joint while the other fingers are being held in extension. Each finger contains two different flexor tendons, FDS and flexor digitorum profundus (FDP). The FDS inserts into the middle phalanx and primarily flexes the finger at the PIP joint. The FDP inserts into the distal phalanx and can flex both the PIP and distal interphalangeal (DIP) joints.
Flexion at the DIP joint is performed by the FDP tendon, so this tendon can be tested by stabilizing the metacarpophalangeal (MCP) and PIP joints in extension, while observing for DIP flexion. Flexion of the PIP joint can arise from both FDS and FDP tendon functions, regardless of whether the MCP joint is in extension or flexion.
Testing for FDS function requires eliminating the contribution of the FDP tendon in order to isolate the FDS. This can be done by stabilizing the other fingers in extension. The FDP has a common muscle belly, so holding the other fingers straight will prevent the FDP from firing. The resultant flexion of the PIP joint will be due to the FDS tendon alone. The FDS tendon has independent muscle bellies. Increased flexion of the DIP joint when the MCP and PIP joints are in flexion is seen in cases of intrinsic tightness.
A 17-year-old boy presents with a mass of the left forearm that has been enlarging over the past 3 months. A photograph is shown. Biopsy of the mass is consistent with spindle cell sarcoma. MRI shows a well-encapsulated mass that does not infiltrate neurovascular structures. CT scan of the chest shows no evidence of metastatic lesions. Which of the following is the most appropriate next step in management? A) Amputation at the midforearm B) Chemotherapy only C) Excision and chemotherapy D) Excision and radiation therapy E) Radiation therapy only
The correct response is Option D.
Soft tissue sarcomas are rare malignant tumors representing less than 1% of all malignancies, with only 25% occurring in the upper extremity. The diagnostic workup generally includes a biopsy, magnetic resonance imaging scan to assess the extent of the tumor and its relationship to adjacent structures, and a computed tomography scan of the chest. This combination allows for appropriate clinical staging of the patient. The role of sentinel lymph node biopsy in the workup of soft tissue sarcomas is controversial. Treatment consists of wide excision, primary reconstruction, and radiation therapy (adjuvant or neoadjuvant). The tumor must be completely removed with a cuff of normal tissue (at least 1 cm); violation of the tumor decreases 5-year survival from 87 to 47%. More than 90% of extremity sarcomas can be managed with a limb-sparing resection.Chemotherapy is rarely indicated and is never used as the only treatment for soft tissue sarcoma. Primary amputation is considered when the tumor infiltrates major neurovascular structures and resection would result in the sacrifice of more than one major peripheral nerve. Additional indications for amputation include involvement of the interosseous membrane, advanced disease with extensive loss of functional tissues, and severe comorbidities limiting reconstructive options.
A 32-year-old man presents to the emergency department for evaluation of a laceration of the right wrist sustained when he punched a glass window 1 hour ago. Physical examination shows a 2-cm transverse laceration of the volar ulnar wrist crease. Wound exploration shows complete laceration of the ulnar nerve. On physical examination of motor function, LOSS of which of the following functions is most likely in this patient?
A) Adduction of the thumb carpometacarpal joint
B) Extension of the metacarpophalangeal joint of the ring and small fingers
C) Extension of the thumb interphalangeal joint
D) Flexion of the interphalangeal joint of the index and middle fingers
E) Flexion of the interphalangeal joint of the ring and small fingers
The correct response is Option A.
The ulnar nerve is the terminal branch of the medial cord of the brachial plexus. It enters the forearm between the two heads of the flexor carpi ulnaris (FCU). In the forearm, the ulnar nerve innervates the FCU and flexor digitorum profundus of the small and ring fingers. It courses distally under the FCU to enter Guyon’s canal at the wrist. The dorsal cutaneous nerve, which gives sensation to the dorsoulnar hand, arises approximately 5 to 7 cm proximal to the ulnar styloid. In Guyon’s canal, the ulnar nerve splits into a deep motor and a superficial sensory branch. The deep motor branch innervates the hypothenar muscles (abductor digiti minimi, opponens digiti minimi, and flexor digiti minimi), as well as the lumbricals to the ring/small fingers, dorsal and palmar interossei, flexor pollicis brevis (deep head), palmaris brevis, and adductor pollicis. The superficial sensory branch in the palm innervates the small finger and the ulnar aspect of the ring finger.
Adduction of the thumb is controlled through activation of the ulnar-innervated adductor pollicis muscle. In the small and ring fingers, extension of the metacarpophalangeal (MCP) joint is performed through activation of the radially innervated extensor digitorum communis and extensor digiti minimi muscles. In the index, middle, ring, and small fingers, flexion of the proximal interphalangeal (PIP) joint is performed through activation of the median-innervated flexor digitorum superficialis.
While flexion of the ring and small finger distal interphalangeal (DIP) joints is produced by the ulnar-nerve innervated FDP tendons to the ring and small finger, the ulnar nerve provides branches to this muscle proximal to this patient’s injury. Extension of the thumb MCP joint is via the radial nerve innervated extensor pollicis brevis muscle.
A 29-year-old man undergoes evaluation for nonunion of a scaphoid fracture. Reconstruction with a vascularized osseous flap is planned, and a medial femoral condyle flap is chosen. During harvest, the vascular pedicle for this flap runs between which of the following structures?
A) Anterior to the tensor fascia lata and posterior to the vastus lateralis
B) Anterior to the vastus medialis and anterior to the adductor tendon
C) Anterior to the vastus medialis and posterior to the rectus femoris
D) Posterior to the rectus femoris and anterior to the vastus lateralis
E) Posterior to the vastus medialis and anterior to the adductor tendon
The correct response is Option E.
The medial femoral condyle osseous free flap has become a useful option for reconstruction of bony defects in the extremities, particularly of the scaphoid waist and proximal pole. The vascular supply to this flap is from the descending geniculate artery in the distal medial aspect of the thigh. To explore and identify the pedicle for this flap, the vastus medialis is reflected anteriorly, and the adductor tendon is found posterior to the vessels. The rectus femoris is located anterior to the dissection for this flap.
A 2-year-old female infant presents with a 3 × 3-cm red, firm, raised mass on the dorsum of the right hand. Physical examination shows a red mass with a smooth surface, and a rim of decreased pigmentation. The patient’s mother reports that the mass has not changed in size, appearance, or coloration since birth. An ultrasound and MRI at age 3 months showed a well-defined, homogenous mass with high- blood flow characteristics. Which of the following is the most likely diagnosis in this patient?
A) Dermatofibrosarcoma protuberans
B) Infantile hemangioma
C) Lymphatic malformation
D) Non-involuting congenital hemangioma (NICH)
E) Venous malformation
The correct response is Option D.
Non-involuting congenital hemangioma (NICH) is a rare form of hemangioma that is present at birth, is stable in size over time (ie, does not involute), and often has a white-grey rim. It is histologically and radiographically similar to infantile hemangioma except that it stains negative for glucose transporter protein 1 (GLUT1).
Venous malformation and lymphatic malformation are low-flow and do not fit the clinical description: they are typically darker in color, amorphous in form, and are compressible. Dermatofibrosarcoma protuberans is rare in children, begins more like a flat scar, and grows over time. It is not a high-flow lesion.
A 32-year-old woman presents to the emergency department with a transverse laceration over the volar nondominant small finger. Physical examination shows full active flexion and extension but pain on resisted flexion. Exploration shows a 20% flexor profundus laceration. Which of the following are the most appropriate joint positions for splinting this patient’s wrist, metacarpophalangeal (MCP) joints, proximal interphalangeal (PIP) joints, and distal interphalangeal (DIP) joints?
A) Wrist extension, MCP joint extension, PIP joint extension, DIP joint extension
B) Wrist extension, MCP joint flexion, PIP joint extension, DIP joint extension
C) Wrist extension, MCP joint flexion, PIP joint flexion, DIP joint extension
D) Wrist flexion, MCP joint extension, PIP joint extension, DIP joint extension
E) Wrist flexion, MCP joint flexion, PIP joint flexion, DIP joint flexion
The correct response is Option B.
The correct position in which to immobilize the hand is intrinsic plus position. This is also known as the safe position that helps to prevent joint stiffness and joint contractures. This goal is to have the collateral ligaments of the wrist, metacarpophalangeal (MCP) joint, proximal interphalangeal (PIP) joint, and distal interphalangeal (DIP) joint at full tension. The wrist is placed between 0 to 30 degrees of extension, the metacarpophalangeal (MCP) joints in 70 to 90 degrees of flexion, and both the PIP and DIP joints into full extension. Since the patient has only a partial-thickness flexor tendon injury, there is no need to consider flexion other than at the MCP joint.
A 40-year-old man presents to the emergency department because of severe pain after sustaining a crush injury to the left lower extremity from a forklift. On physical examination, the lower leg is tense and swollen circumferentially. Sensation to the foot is diminished. Distal pulses are palpable. X-ray study does not show any fractures. Which of the following is the most appropriate next step in management? A) Ace wrap compression B) CT angiography C) Emergent fasciotomy D) MRI E) Observation and leg elevation
The correct response is Option C.
The patient displays the signs and symptoms of acute compartment syndrome, a surgical emergency requiring emergent fasciotomy. Acute compartment syndrome requires prompt diagnosis and expeditious treatment in order to minimize morbidity.
Compartment syndrome can occur following a substantial soft tissue crush injury, even in the absence of a fracture, such as in this clinical scenario. Severe pain is usually the presenting complaint. It may be out of proportion to the injury and unresponsive to analgesics. The presence of paresthesias can signify nerve hypoxia from elevated compartment pressures. Pallor, paralysis, and pulselessness are very late signs. Nerve and muscle do not tolerate long periods of ischemia and may undergo irreversible damage if surgical decompression is delayed.
Compartment syndrome is primarily a clinical diagnosis, but measurement of compartment pressures can provide additional information especially if the diagnosis of compartment syndrome is less obvious. If compartment pressures are greater than 30 mmHg or if the differential pressure (difference between diastolic blood pressure and compartment pressure) is less than 30 mmHg, then fasciotomy is recommended.
Observation and leg elevation would not be appropriate management in the setting of acute compartment syndrome. CT angiography would not be indicated in this case, where there is a low suspicion of vascular injury. MRI has been used in the diagnosis of chronic exertional compartment syndrome but has little value in the setting of acute trauma.
A 56-year-old woman with a traumatic defect of the upper third of the tibia undergoes open reduction and internal fixation with tibial nail. Soft tissue coverage with a gastrocnemius flap is planned. Which of the following arteries provides the dominant blood supply for this flap? A) Anterior tibial B) Peroneal C) Popliteal D) Posterior tibial E) Sural
The correct response is Option E.
Each head of the gastrocnemius muscle is supplied by the sural artery: either the medial sural or lateral sural artery for medial and lateral gastrocnemius, respectively. The arteries arise from the popliteal artery about 3-4 cm above the head of the fibula and enter the medial and lateral heads of the gastrocnemius at about the level of the head of the fibula. The flap can be rotated to cover soft-tissue defects of the anterior distal aspect of the knee. The flap ranges from 5 to 9 cm in width and from 13 to 20 cm in length. It provides a vascular bed for a skin graft and improves the delivery of oxygen and systemic antibiotics. The other listed arteries do not supply the gastrocnemius muscles.
A 9-month-old boy presents for evaluation of unilateral preaxial polydactyly. X-ray studies show triphalangism of the accessory thumb. Which of the following is the most likely Wassel classification in this patient? A) Type II B) Type III C) Type IV D) Type V E) Type VII
The correct response is Option E.
Preaxial polydactyly describes patients with thumb duplication. In 1969, Wassel (as the fellow of Adrian Flatt) described a categorization system for radial polydactyly corresponding to the level of skeletal duplication. There have been many subsequent modifications of this classification system, but this remains the most commonly used. Type IV (duplicated proximal and distal phalanges) is the most common duplication, followed by Type II (duplicated distal phalanx). A Type VII duplication involves a triphalangeal thumb and is the only deformity in which there is triphalangism.
A 65-year-old woman presents with severe osteoarthritis of the proximal interphalangeal (PIP) joint of the nondominant left middle finger. Medical history includes chronic pain and an angular deformity of the joint. Range of motion of the PIP joint is 30 to 60 degrees. Silicone implant arthroplasty is planned. Which of the following is the principle benefit of this procedure? A) Correction of angular deformity B) Improved cosmesis C) Improved range of motion D) Increased grip strength E) Pain relief
The correct response is Option E.
Expected outcomes for small joint implant arthroplasty are pain relief with similar range of motion to preoperative values. The procedure involves excision of the arthritic proximal phalanx head and middle phalanx base and replacement with a silicone stemmed implant. The implant acts as a spacer for development of a scar capsule.
Although angular deformity is corrected with this procedure, and many patients report satisfaction with the improved appearance of the alignment of the finger, the primary goal of the procedure is pain relief from underlying arthritis. Outcome studies have not demonstrated improved grip strength or range of motion. Long-term outcome studies show 90% implant survival at 10 years, high patient satisfaction, and a low revision rate despite a relatively high incidence of implant fracture or deformity over time.
A 6-year-old boy presents with a supracondylar fracture sustained during a fall on an outstretched hand. A splint with the elbow flexed less than 90 degrees is placed. The patient is screaming in pain. Examination shows the affected hand has a 3- second capillary refill. Which of the following is the most appropriate next step in management?
A) Closed reduction
B) Continued observation and application of ice packs
C) Elevation of the arm
D) Exploration of the brachial artery
E) Replacement of the current splint with an elbow extension splint
The correct response is Option A.
Supracondylar fractures are one of the most common traumatic fractures seen in children. It occurs most commonly in children 5 to 7 years of age with similar male and female incidence. The mechanism is usually from a fall onto an outstretched hand. The fracture can lead to severe forearm edema, then ischemia leading to Volkmann’s contracture.
Immobilization would be long arm casting with the elbow flexed at less than 90 degrees. Arm elevation would decrease tissue perfusion and would therefore be contraindicated. Immediate bedside closed reduction by gentle traction and elbow flexion to 20 to 40 degrees would be indicated in this case as a next step. If the closed reduction is unsuccessful or ischemia persists after reduction or recurs, urgent operative closed reduction with percutaneous pinning is required. Pins are placed to prevent recurrence. Brachial artery exploration could be required if ischemia has not resolved even after successful reduction, but not initially.
A 40-year-old woman presents with small, non-healing ulcers of the right index and middle fingertips. Medical history includes limited scleroderma diagnosed 5 years ago, chronic pain, and color changes of the fingers in cold temperatures. The patient's symptoms have not improved with administration of nifedipine. Angiography shows diffuse vascular narrowing without any focal lesions. Which of the following is the most appropriate intervention for pain relief and ulcer healing in this patient? A) Cervical sympathectomy B) Continuous brachial plexus blockade C) Digital bypass D) Onabotulinum toxin A E) Stellate ganglion block
The correct response is Option D.
This patient has Raynaud’s phenomenon associated with scleroderma. The pathophysiology of Raynaud’s is thought to be related to sympathetic hyperactivity, elevated plasma endothelin, increased peripheral alpha-2 receptors, and possibly abnormal platelet and red cell function. Botulinum toxin type A has been shown to improve digital perfusion on laser Doppler, decrease pain, and result in ulcer healing. In a series of 33 patients injected with 50 to 100 U of onabotulinum toxin A, all patients had ulcer healing by 60 days postinjection. Pain relief typically occurred within 5 to 10 minutes of injection and complication rates were low and limited to injection site reactions. A prospective, randomized, placebo-controlled trial showed patients with limited scleroderma and shorter duration of disease had the best response to onabotulinum toxin A.
Stellate ganglion blocks have been shown to have only variable success for Raynaud’s with only short-term symptom relief and no effect on ulcer healing. Stellate blocks may not disrupt all sympathetic input to the extremity. Brachial plexus blocks may help with perfusion temporarily but are advocated mainly in patients undergoing microvascular surgery. Their use is not recommended in this setting. Surgical bypass to the superficial palmar arch has been shown to increase blood flow to the hand and improve ulcer healing. However, bypass to the digital vessels would not be indicated as the distal target vessels are often diminutive without adequate flow.
Which of the following cranial nerves develops with the first branchial arch? A) Facial (VII) B) Glossopharyngeal (IX) C) Hypoglossal (XII) D) Trigeminal (V) E) Vagus (X)
The correct response is Option D.
The first branchial arch is associated with the trigeminal nerve (V), second branchial arch with the facial nerve (VII), third branchial arch with the glossopharyngeal (IX), fourth branchial arch with the superior laryngeal (X), and sixth branchial arch with the recurrent laryngeal (X).
A 23-year-old woman with severe progressive hemifacial atrophy that has been stable for 3 years now desires a long-term stable reconstruction. Which of the following is the most appropriate recommendation for reconstruction of this patient’s facial asymmetry?
A) Alloplastic bony augmentation
B) Contralateral suction lipectomy
C) Delay reconstruction until it has been stable for 10 years
D) Free tissue transfer
E) Hyaluronic acid injections
The correct response is Option D.
Progressive hemifacial atrophy is also known as Parry-Romberg syndrome. The progression is ultimately self-limiting. Reconstruction 2 years or more after burn out is commonly accepted. For very mild asymmetry, hyaluronic acid fillers can improve symmetry, but require recurrent treatments. For mild to moderate asymmetry, microfat grafting can restore symmetry. Multiple sessions may be required to achieve long-term correction. For severe asymmetry, free muscle flap with parascapular flap or anterolateral thigh flap can provide enough soft tissue bulk for long-term correction.
Alloplastic bony augmentation would correct any potential bony deficiencies, but would not address any soft tissue deficiencies.
A 25-year-old man presents with a diagnosis of bruxism and pain. Which of the following drugs is the most appropriate for treatment? A) Amitriptyline B) Botulinum toxin type A C ) Clonidine D) Escitalopram E) Fluoxetine
The correct response is Option B.
Botulinum toxin A has shown efficacy in the treatment of pain symptoms from bruxism and, thus, its effects are not limited to improving masseter hypertrophy. It has shown equal efficacy when compared with occlusal splints.
Botulinum toxin A has been shown to be more effective than placebo in decreasing pain from bruxism by a patient report using the visual analogue scale (VAS). However, other pharmacotherapies, such as clonidine and amitriptyline, have demonstrated no improvement
in pain relief over placebo.
Bruxism is a reported side effect of selective serotonin reuptake inhibitors such as fluoxetine and escitalopram.
A 15-year-old girl has a 12-month history of pain and fullness in the right supraorbital rim. She has café-au-lait spots, a history of precocious puberty, and a recent pathologic rib fracture. Which of the following is the most likely pathology of the lesion? A) Dermoid cyst B) Fibrous dysplasia C) Neurofibroma D) Osteoblastoma E) Rhabdomyosarcoma
The correct response is Option B.
This patient has a classic presentation of McCune-Albright syndrome. Patients with McCuneAlbright present with a triad of polyostotic fibrous dysplasia, precocious puberty, and skin pigmentation (eg, café au lait spots). Additionally, they may have hyperfunctioning
endocrinopathies such as growth hormone excess. If these patients present with intramuscular myxomas, it is known as Mazabraud syndrome. Malignant degeneration of fibrous dysplasia has been reported in up to 4% of patients with McCune-Albright syndrome. Management depends on the clinical presentation and functional impact of the lesions, and is primarily surgical.
Patients with neurofibroma may have café au lait spots, but not precocious puberty or pathologic fractures. Dermoid cysts generally do not present with pain and are often noted at a much younger age than the patient described.
A patient with Möbius syndrome is referred for facial reanimation with free gracilis muscle flaps. Which of the following donor nerves has the lowest morbidity and is
used most often for this type of reconstruction?
A) Contralateral facial
B) Glossopharyngeal
C) Hypoglossal
D) Masseteric
E) Spinal accessory
The correct response is Option D.
In cases of facial palsy where the facial nerve is unavailable for use as a donor nerve, nerve transfers are the best option. Of the local options, the masseteric nerve provides the most appropriate and most commonly used transfer because of its proximity and low morbidity when harvested.
While the hypoglossal nerve is sometimes used as a donor nerve for ipsilateral facial reanimation, the process of harvesting can be associated with considerable oropharyngeal morbidity because of ipsilateral tongue atrophy. In addition, the patient in this scenario requires a bilateral facial reanimation procedure, and harvesting of both hypoglossal nerves would paralyze the tongue.
A cross-facial nerve transfer is not an available option in this scenario because the patient has Mobius syndrome with bilateral facial nerve agenesis. Use of bilateral glossopharyngeal nerves as donors would create significant oropharyngeal function morbidity, and, even in cases of unilateral facial reanimation, is not commonly used. Use of both spinal accessory nerves as donor nerves in this case would create significant morbidity. Even in a case of unilateral facial reanimation, use of the masseteric nerve is a significantly better choice.
A 14-year-old girl with Crouzon syndrome presents with a severe Angle Class III malocclusion, mid face retrusion, and severe sleep apnea. She is scheduled to undergo Le Fort III advancement using distraction osteogenesis. The risk for complications with this procedure is closest to which of the following? A) 5% B) 20% C) 40% D) 60% E) 80%
The correct response is Option B.
There are several important advantages of distraction osteogenesis for Le Fort III advancement versus conventional single-stage advancement with bone
grafting and these include: less regression, greater advancement distance, and no need for bone grafting. Le Fort III distraction is not without its issues. Major and minor complications have been reported in nearly 20% of patients undergoing this procedure; these complications include bone loss, pin migration, loss of fixation, meningitis, seizures, and cerebrospinal fluid leaks. Several recent reports show that these complications occur in approximately 20% of cases.
A 50-year-old woman has ptosis of the right upper eyelid 4 days after receiving botulinum toxin for cosmetic treatment of corrugator rhytides. Stimulation of which
of the following muscles is most likely to improve her eyelid function?
A) Iris sphincter
B) Levator palpebrae superioris
C) Müller
D) Orbicularis oculi
E) Procerus
The correct response is Option C.
Iopidine and phenylephrine are both adrenergic agonists, which selectively increase eyelid elevation in the absence of levator function. They do this by selectively targeting α2 adrenergic receptors on smooth muscle, which results in the contraction of Müller muscle.
This muscle adjoins onto the levator palpebrae superioris muscle and on contraction, causes moderate eyelid elevation. Although it does not have the level of eyelid elevation as the levator muscle, Müller muscle can still give partial improvement and decrease ptosis until the
botulinum toxin has dissipated. The orbicularis oculi, procerus, and iris sphincter muscles are not involved in eyelid closure. The levator palpebrae muscle is the injured muscle from the botulinum toxin and there is no reversal drug available.
A 35-year-old woman is unhappy with the appearance of her “square face,” especially at the bottom jaw “near the corners,” and she wishes to have a smoother mandible contour permanently. Which of the following is the most appropriate treatment? A) Alloplastic implant B) Autologous fat grafting C) Hyaluronic acid dermal filler D) Mandible angle and body contouring E) Suction lipectomy
The correct response is Option D.
Mandible contouring surgery, frequently called mandible angle reduction, is a bony procedure to decrease the angular contours in a “square face” or bottom jaw with “sharp corners.” The term “mandible angle” reduction is a misnomer, as usually both the mandible angle and the
mandible body need to be gracefully contoured or resected to result in an aesthetically pleasing, rounder face.
Although fillers, fat grafting, and implants are used in the face, they are not usually used for mandible contouring. Suction lipectomy for a patient with a square jaw would not be successful.
The stylopharyngeus muscle is innervated by which of the following nerves? A) Facial (VII) B) Glossopharyngeal (IX) C) Hypoglossal (XII) D) Trigeminal (V) E) Vagus (X)
The correct response is Option B.
The pharyngeal muscles are all innervated by the vagus (X) nerve, except the stylopharyngeus muscle, which is innervated by the glossopharyngeal nerve (IX).
The trigeminal nerve (V) is responsible for facial and oral sensation. The maxillary branch (V2) is responsible for sensation of the upper teeth, upper lip, hard palate, cheeks, and nasopharyngeal mucosa. The mandibular branch (V2) provides sensory fibers for the lower teeth, lower mucosa of the mouth and the anterior two-thirds of the tongue. The facial nerve (VII) provides motor innervation of the muscles of facial expression and the posterior bellies of the stylohyoid and digastric muscles. The vagus nerve (X) provides motor innervation to
all of the pharyngeal muscles except the stylopharyngeus muscle. The hypoglossal nerve (XII) provides motor innervation to the intrinsic and extrinsic tongue muscles and also provides motor innervation to the geniohyoid muscle through the ansa cervicalis.
A newborn presents with an asymmetric bilateral cleft lip-nose-palate; the right side is incomplete and the left side is complete. A photograph is shown. Which of the following sequences of repair is most likely to provide the optimal outcome?
A) One-stage repair of the bilateral cleft lip
B) Repair cleft palate at the initial surgery
C) Repair the complete side cleft lip first, then stage incomplete side cleft lip at a later surgery
D) Repair the incomplete side cleft lip first, then stage complete side cleft lip at a later surgery
E) Simultaneous repair of cleft lip and cleft palate
The correct response is Option A.
Bilateral cleft lip repair is more commonly symmetric and is usually repaired as a one-stage repair around 3 to 6 months of age.
Asymmetrical bilateral cleft lip repair can be done in one stage or two stages, and the sequencing has been controversial. However, two recent studies show convincing evidence that even for asymmetrical bilateral cleft lip repairs, a one-stage repair leads to overall better symmetry and cleft lip repair outcomes. Therefore, a two-stage bilateral cleft lip repair is incorrect.
Cleft palate repair is typically performed closer to 9 to 12 months of age, therefore, cleft palate repairs at 6 to 9 months of age are less typical and irrelevant, as the two-stage repair is considered not optimal for this asymmetric bilateral cleft lip scenario.
Which of the following head and neck pathologies does NOT have the potential to transform into an entity that can metastasize? A) Ameloblastoma B) Basal cell carcinoma C) Bowen disease D) Pleomorphic adenoma E) Warthin tumor
The correct response is Option E.
Warthin tumors are generally located in the parotid gland; they do not transform into a malignant tumor and, thus, remain benign.
Bowen disease is a very early form of skin cancer that is easily treatable and is also referred to as squamous cell carcinoma in situ. The tumor is usually very slow-growing, but there is a small chance it could turn into squamous cell skin cancer if it is left undiagnosed or
neglected. There is a known metastatic rate of this cancer.
Carcinoma ex pleomorphic adenoma is a carcinoma that arises from a primary or recurrent benign pleomorphic adenoma. It can metastasize and is highly lethal. Luckily, it is a rare cancer.
Basal cell carcinoma rarely causes metastases. There are now drugs available to treat this disease, such as hedgehog inhibitors.
Ameloblastoma is a rare odontogenic epithelial tumor that represents only approximately 1% of all jaw tumors, but it is the second most common odontogenic tumor. However, a histologically benign-appearing ameloblastoma can metastasize to local lymph nodes or other distant organs, such as the brain, lung, skin, etc.
A 19-year-old right-hand–dominant man presents with an injury to the right ring finger sustained when he hung on the basketball rim after slam dunking a basketball 1.5 weeks ago. Photographs are shown. X-ray studies are unremarkable. Which of the following is the most appropriate next step in management?
A) Fusion of distal interphalangeal joint in 20 degrees of flexion
B) Primary repair
C) Staged tendon reconstruction
D) Tenodesis
E) Observation
The correct response is Option B.
This patient has an avulsion of the flexor digitorum profundus (FDP) tendon from its insertion at the volar base of the distal phalanx (Zone 1), also known as a jersey finger. The ring finger is involved in 75% of cases of this type due to its prominence during grip (longer than adjacent fingers during grip in most patients). Leddy and Packer classify jersey finger injuries as Types 1 through 5:
Type 1: Avulsion with retraction into the palm with disruption of the vincular blood supply,
Type 2: Retraction to the level of the proximal interphalangeal (PIP) joint,
Type 3: Associated fracture fragment holds tendon at distal interphalangeal (DIP) joint lever,
Type 4: Fracture with tendon avulsion from bony segment,
Type 5: Fracture, avulsion, with distal phalangeal comminution.
Since there was no bony fragment on x-ray study, this is either a Type 1 or 2 and an attempt at primary repair up to 3 weeks after injury is almost invariably achievable. This offers the best possible outcome versus the other options, which are either salvage or not practical. Observation is a poor alternative as the loss of the FDP can result in DIP joint hyperextension and can adversely affect the ring finger mobility and function. Fusion is a salvage procedure and should not be considered when repair is possible. Staged repair could be considered in a more chronic injury but is tedious and has variable outcomes. It is unnecessary in this patient. Tenodesis is another salvage procedure that has been described in unrepairable injuries.
A 12-year-old boy presents with warts on his fingers. His parents report that the warts have persisted despite application of a variety of topical treatments. Which of the following viruses is the most likely cause of this patient's condition? A) H1N1 B) Hepatitis C C) Herpes simplex D) Human immunodeficiency E) Human papilloma
The correct response is Option E.
The cause of warts is the human papilloma virus (HPV). There are more than 100 known types of HPV. Types 2 and 4 are the most common cause of warts on the hands, as in this patient. People whose occupations expose their hands to wet environments, such as meat, poultry, and fish handlers and veterinary surgeons have higher risk of developing warts. The virus can be transmitted on shared clothing or public spaces, such as showers, and then gain entry through breaks in the skin. The virus is then able to alter the squamous epithelium to produce a cauliflower-like growth. Warts can be present in single lesions or multiple lesions. They are often painless and are usually not cancerous but can be a source of embarrassment for the patient. Warts are generally self-limited and will resolve spontaneously within months or years. Spontaneous resolution appears to occur in 50% of children within 1 year and 90% within 5 years.
There are many treatments for warts. Home remedies include topical salicylic acid, duct tape, and cold treatments. Clinical treatments have shown topical acids and cryotherapy to be equally effective. Intralesional injections and topical antivirals, as well as immunologic stimulators of interferon, have been tried with some success, including purified
candida; measles, mumps, and rubella; and tuberculin (PPD) protein. Chemical ablation with silver nitrate has been shown to resolve almost half of warts a month after a 9-day treatment protocol. Mechanical removal can be performed with direct excision with good
success. Pulsed dye and CO2 lasers have been successfully used to ablate warts but can be painful, expensive, and leave scars. Periungal warts can be particularly difficult to treat topically and may require a more invasive treatment method. A patient’s ability to eradicate a wart can be reduced by a compromised immune system. Multiple progressive warts in immunosuppressed patients may need to be biopsied as these warts may transform into squamous cell carcinoma.
Human immunodeficiency virus (HIV), H1N1, hepatitis C, and herpes simplex are all viruses that affect humans but do not cause lesions on the fingers. Herpes simplex virus is the cause of cold sores around the mouth and genital herpes. Hepatitis C is a virus that causes inflammation of the liver. HIV decreases the effectiveness of the immune system. H1N1 is an influenza virus and known cause of the “swine flu” outbreak in 2009.
A 41-year-old woman who is a dentist comes to the office because she has had intense burning and pruritus of the dominant index finger for the past 6 days. The patient reports a small vesicular rash on the finger that has since progressed to form a small blister. She has had intermittent fever and malaise. Which of the following is the most appropriate management?
A) Incision and drainage of the bullae
B) Intravenous administration of cefazolin
C) Oral administration of acyclovir
D) Topical application of silver sulfadiazine
E) Observation only
The correct response is Option E.
The patient has a history and physical findings consistent with herpetic whitlow. This is a viral infection caused by herpes simplex virus and is more common in medical and dental personnel. Tzank smear or antibody titers can confirm a diagnosis but are unnecessary in the management of this patient. Treatment is primarily nonoperative and involves observation, as the course of the illness is self limiting with resolution in 1 to 3 weeks. Intravenous antibiotics would not treat this viral infection. Incision and drainage is unnecessary and may lead to a bacterial superinfection or systemic dissemination of herpes simplex virus. Acyclovir or valacyclovir may shorten the duration of symptoms, but must be started within 2 to 3 days of onset. Topical application of an antimicrobial would provide no benefit in this case. Surgical drainage of the bullae should not be performed because it may increase the risk of spreading the herpes virus and may also lead to bacterial superinfection.
A 53-year-old African American man is referred for evaluation of a 3-mm-wide pigmented streak of the left index fingernail. A full-thickness biopsy of the nail matrix confirms the diagnosis of melanoma. Which of the following factors is most pertinent in determining prognosis and 5-year survival in this patient? A) Mitotic rate B) Tumor free resection margins C) Tumor location D) Tumor stage E) Width of the lesion
The correct response is Option D.
This patient has acral lentiginous melanoma (ALM) based on the clinical description of a dark- skinned man with a tumor on the fingernail presenting as a pigmented streak. The prognosis for ALM is typically worse than other melanoma subtypes. The poor survival rate of these patients may be due to a delay in diagnosis. As in other melanoma subtypes, tumor thickness is the most important prognostic indicator. Overall, 5-year survival for ALM is 80 versus 91% for all melanomas. Acral lentiginous melanoma is the least common subtype of melanoma, however, it makes up the highest percentage of cutaneous melanomas in dark-skinned patients. ALM is predominantly found on the palms, soles, and nail beds. This is in contrast with other melanoma subtypes that typically occur in sun-exposed areas. However, the location of the tumor does not directly influence the prognosis. It is more correlated to diagnosis at a later stage.
Mitotic rate and other pathologic characteristics such as microscopic ulceration, lymphatic, or nerve involvement can upstage the tumor. However, it is not the primary determinant of tumor stage, and mitotic rate is no longer considered as part of staging in early melanomas.
Achieving adequate resection margins in ALM may be difficult, especially in tumors involving the nail unit. Amputation at the next most proximal joint is often recommended. Regardless of the status of the margins, prognosis is still determined by depth at diagnosis. Thicker tumors have a higher incidence of nodal involvement and metastatic disease. In later stage disease, surgical resection of the primary tumor is for diagnostic purposes, local control, and occasionally palliative care.
Pigmented lesions of the nails greater than 2 mm have a higher likelihood of being invasive melanoma, but width of the lesion is not involved in tumor staging or prognosis.
In a transhumeral amputee, targeted muscle reinnervation can be utilized to improve control in a myoelectric prosthesis. Which of the following nerve transfers can be performed to provide intuitive prosthetic control for hand closure?
A) Median nerve to short head of biceps
B) Musculocutaneous nerve to long head of biceps
C) Radial nerve to lateral head of triceps
D) Radial nerve to long head of triceps
E) Ulnar nerve to lateral head of triceps
The correct response is Option A.
Targeted muscle reinnervation (TMR) utilizes a set of nerve transfers in order to allow intuitive prosthetic control for upper extremity amputees. Functioning nerves that no longer have their distal muscle target can be transferred to intact proximal muscles and generate a novel electrical signal that can be picked up by a myoelectric prosthesis. Another benefit of TMR is the potential to prevent or treat painful neuromas.
In the case of a transhumeral amputee, elbow flexion myoelectric prosthetic control is maintained by preserving musculocutaneous innervation to the long head of the biceps muscle. The distal remnant of the median nerve is transferred to the motor nerve of the biceps short head to create a signal for prosthesis hand closure. Elbow extension signals are maintained with radial innervation of the long head of the triceps. Signals for prosthesis hand opening are created with transfer of the distal radial nerve to the motor nerve of the triceps lateral head.
A 2-year-old male infant presents with a congenital deformity of the ring finger. A photograph and x-ray study are shown. The patient's parents report that the condition limits his ability to make a full fist but is otherwise painless. Which of the following is the most likely diagnosis in this patient? A) Amniotic band syndrome B) Camptodactyly C) Congenital trigger finger D) Macrodactyly E) Syndactyly
The correct response is Option A.
Amniotic band syndrome (ABS) has an incidence of 1/1200 to 1/15,000 births. Some congenital anomalies have been associated with ABS including cleft palate, imperforate anus, equinovarus, and body wall defects.
The etiology of ABS has two theories. The intrinsic defect theory endorses genetic abnormalities which lead to mesenchymal hypoplasia and scarring. The extrinsic theory endorses amniotic tissue which entangles fetal parts leading to constriction.
Constriction varies leading to a spectrum of clinical manifestations from skin dimpling to digital amputation. Neurovascular injury can manifest as peripheral nerve palsy, lymphedema and arterial insufficiency. Syndactyly is common, and acrosyndactyly is pathognomonic of ABS. The Patterson classification system has four subtypes. The first is a simple constriction ring. The second has a constriction ring that affects the digit distal to the ring, with or without lymphedema. The third consists of constriction rings with acrosyndactyly. The fourth is characterized by amputation at any level.
Camptodactyly is defined as a painless and progressive non-traumatic contracture of the proximal interphalangeal (PIP) joint. It affects around 1% of the population and the great majority of the cases are extremely mild and asymptomatic. The cause of the contracture is controversial. There have been descriptions of malformations of the superficial flexor of the fingers, lumbrical muscles, and the transverse and oblique retinacular ligaments. There may also be alterations to the configurations of the PIP joint.
Congenital trigger finger differs from congenital trigger thumb. Congenital trigger finger is rare and presents more commonly in the ulnar digits with associated malformations of the superficial and deep flexors. It presents with sporadic locking. Release of the A1 pulley alone is not adequate, with tenoplasty of the chiasm and partial opening of the A2 pulley generally necessary.
Syndactyly is a variable fusion between two adjacent fingers, and is one of the most common congenital
deformities, occurring in 1:2000 live births.
Classification includes:
Simple: fusion only through the skin
Complex: bone connection.
Complete: the entire commissure is involved, including the nail bed Incomplete: the nail bed is not involved.
Complicated: involvement of vascular tissues, tendons or nerves.
It can occur separately or as a manifestation of a syndrome, such as Streeter, Apert or Poland, in which the severity of the syndactyly is more significant.
Macrodactyly is a congenital overgrowth disorder and represents 0.9% of upper extremity congenital anomalies. Digital enlargement involves all tissue types and maintains patterns of growth and anatomic relationships within the affected portion of the hand.
The term “macrodactyly” is reserved for nonsyndromic, congenital enlargement of a digit or digits that occurs in isolation without associated limb hemihypertrophy or vascular anomaly.
In patients with rheumatoid arthritis, the inciting event in development of a boutonniere deformity is which of the following?
A) Central slip attenuation
B) Intrinsic tightness
C) Lateral band volar subluxation
D) Oblique retinacular ligament contracture
E) Proximal interphalangeal (PIP) joint synovitis
The correct response is Option E.
Rheumatoid arthritis is a chronic autoimmune disease characterized by inflammation and deterioration of the joints. Synovial proliferation is the hallmark of rheumatoid arthritis and is often seen early in the course of the disease. There is a progression to synovial pannus formation, periarticular bone demineralization, cartilage destruction, and subchondral osseous erosions. This process is mediated by synovial infiltration of activated T lymphocytes, which promote chronic synovial inflammation.
The boutonniere deformity is extremely common in patients with rheumatoid arthritis. It is characterized by flexion of the proximal interphalangeal (PIP) joint with hyperextension of the distal interphalangeal (DIP) joint.
The causative event of boutonniere deformity in rheumatoid arthritis is synovitis and synovial pannus formation within the PIP joint. This causes the joint capsule to distend, resulting in attenuation of the central slip. Central slip insufficiency results in loss of PIP joint extension and subsequent volar translocation of the lateral bands, which further accentuates the deformity by providing a flexion force across the PIP joint. Extension forces are transferred to the DIP joint. Contraction of the oblique retinacular ligament is associated with a fixed deformity.
Intrinsic tightness would cause the PIP joint to be unable to be flexed when the MP is in extension.
A 23-year-old man presents with painful, red swelling over the dorsum of the right middle finger metacarpophalangeal (MCP) joint 3 days after he punched someone in the face during a bar fight. The patient reports decreasing range of motion of the hand and inability to grasp objects. He was previously seen in the emergency department the night of the injury and x-ray studies were negative for fracture or foreign body. Which of the following is the most likely causative organism? A) Clostridium perfringens B) Eikenella corrodens C) Pasteurella multocida D) Pseudomonas aeruginosa E) Staphylococcus epidermidis
The correct response is Option B.
Eikenella corrodens is an anaerobic organism present in human oral flora and has been associated with human bite wounds. Group A Streptococcus is also a common pathogen in a fight bite injury like the one this patient has.
This patient has most likely sustained a “fight bite,” which results from tooth penetration of the metacarpophalangeal (MCP) joint after striking someone in the mouth with a clenched fist. These injuries can often be underappreciated, as the underlying defect in the extensor hood and joint capsule may not be seen on examination when the fingers are extended during examination in an emergency department. The joint can become contaminated with oral flora. Penetrating injury with high bacterial load can result in a septic joint and lead to destruction of cartilage and osteomyelitis. Recreation of the flexed fist position may help in lining up the structures and assist in identification of the injury. Treatment is aggressive antibiotic therapy and surgical exploration with irrigation and debridement of the joint to remove debris.
Clostridium perfringens is a gram-negative rod associated with gas gangrene, which results in subcutaneous crepitus and can be rapidly progressive. Pasteurella multocida is a gram- negative anaerobic bacterium most commonly associated with cat bite
infections. Pseudomonas aeruginosa is a gram-negative rod that can be associated with diabetic wound infections. And Staphylococcus epidermidis is a gram-positive cocci present on the skin. It has been associated with implant infections. None of these pathogens are as likely to be present in a fight bite as Eikenella corrodens.
A 30-year-old man presents to the emergency department with acute left wrist pain after falling 10 feet from a ladder. X-ray studies of the left wrist are shown. After failed closed reduction, the patient reports tingling that progresses to worsening and constant numbness of the left index and long fingers over the course of 6 hours. Which of the following urgent interventions is most appropriate? A) Aspiration of the wrist B) Carpal tunnel release C) MRI of the wrist D) Open reduction of the scaphoid E) Repeat closed reduction
The correct response is Option B.
This patient has a type IV perilunate dislocation, or a true lunate dislocation. This represents a complete disruption of the ligamentous stabilizers about the lunate. These injuries are high energy and can be ligamentous only (lesser arc injuries) or include fractures (greater arc) and are then termed perilunate fracture dislocations. Mayfield et al described the stages of injury progressing from radial to ulnar in a type IV dislocation, including injury of the scapholunate ligament, disruption of the lunocapitate joint, injury of the lunotriquetral ligament, and dislocation of the lunate from its fossa at the radiocarpal joint volarly into the carpal tunnel.On posteroanterior x-ray study of the wrist, there will be disruption of Gilula’s lines. On lateral x-ray study, a “spilled teacup” sign is seen.
Closed reduction with relaxation and traction is important, as the lunate needs to be relocated to its fossa to restore relative alignment of the wrist and to decompress the median nerve in the carpal tunnel. Surgical intervention can then be performed for open reduction of the joints and ligament repair after swelling has improved. However, progression in median nerve symptoms in the setting of successful or failed closed reduction is indicative of acute carpal tunnel syndrome and necessitates urgent surgical intervention.
Advanced imaging such as MRI is not required but may be helpful. Repeat closed reduction is likely to fail at this time, may worsen the swelling, and is unlikely to resolve the carpal tunnel symptoms. Open reduction of the scaphoid is not emergent, and the patient does not have a scaphoid fracture. Aspiration of the wrist will not resolve the inciting etiology of the patient’s carpal tunnel symptoms.
Resistance to which of the following maneuvers is most likely present in a digit that has intrinsic tightness?
A) Passive extension of the metacarpophalangeal (MCP) joint with the proximal interphalangeal (PIP) joint held in hyperextension
B) Passive extension of the PIP joint with the MCP joint held in hyperflexion
C) Passive flexion of the DIP joint with the PIP joint held in hyperextension
D) Passive flexion of the PIP joint with the MCP joint held in hyperextension
E ) Passive flexion of the PIP joint with the MCP joint held in hyperflexion
The correct response is Option D. The intrinsic muscles (dorsal/palmar interossei and lumbricals) are responsible for much of the fine motor function of the hand. Contractures of these muscles lead to a loss of the delicate and complex balance of the intrinsic and extrinsic muscles and typically results in the clinical picture of an intrinsic-plus hand. The intrinsics attach to the extensor mechanism through the lateral bands and facilitate force transmission from the muscles to the extensor mechanism on the proximal and distal phalanges. Because of their line of pull, the intrinsics are responsible for metacarpophalangeal (MCP) joint flexion and proximal interphalangeal (PIP) joint extension. The intrinsic tightness test (i.e. Bunnell test) requires one to assess passive PIP joint flexion with the MCP joint extended. This is compared with passive PIP joint flexion with the MCP joint in flexion which assesses for extrinsic tightness. If there is a substantial increase in resistance to PIP joint flexion with the MCP joint in extension, then the test is considered positive and indicative of intrinsic tightness or adhesions of the lateral bands. Trauma is the most common cause of intrinsic muscle contracture. Spasticity from an upper motor neuron lesion (e.g. traumatic brain injury, cerebrovascular accident, cerebral palsy) may also lead to intrinsic contracture. Arthritis may also lead to intrinsic contracture resulting from joint deviation or dislocation. In trauma, initial treatment is directed at edema prevention and aggressive hand therapy. Patients with spasticity from an upper motor neuron disorder are also initially managed with therapy and splinting. If these modalities are insufficient, surgical release of the intrinsic muscles or tendons (proximal or distal depending on extent of involvement) may improve posture and function. Ulnar motor neurectomy is another option in severely affected individuals to decrease intrinsic muscle tone and improve posture and function, but is only effective in the absence of a fixed MCP joint contracture.
Which of the following failures in embryologic development is most likely to have caused the deformity pictured in the photographs shown?
A) Differentiation of the zone of polarizing activity
B) Formation of the apical ectodermal ridge
C) Inhibition by en-1
D) Longitudinal formation
E) Programmed cell death
The correct response is Option E. The hand plate initially forms with webbed digits. In order to have separate fingers, the interdigital tissue must undergo programmed cell death/apoptosis. Bone morphogenetic protein (BMP) plays an essential role in this process. A complex interplay creates failure of longitudinal formation (i.e., radial club hand). Removal of the apical ectodermal ridge (AER) results in limb truncation. The zone of polarizing activity (ZPA) is present in the posterior aspect of the developing upper limb and helps direct theanteroposterior axis (radial-ulnar axis). Sonic hedgehog (SHH) is the critical signaling factor. ZPA transplantation or excess SHH results in mirror hand deformity. The dorsoventral axis is another pathway critical for appropriate limb development. The WNT7A signaling molecule is expressed in the developing upper limb dorsal ectoderm, activating the LIM homeodomain, resulting in the expression of LMX1B transcription factor from the dorsal mesenchyme, whereas the ventral ectoderm induces the expression of en-1. These signaling factors are necessary for the formation of dorsal versus palmar structures of the hand.
A 35-year-old man presents for evaluation of a laceration to the lateral aspect of the right lower leg 5 cm distal to the knee that he sustained when he fell from a bicycle 2 months ago. Findings on electromyography and nerve conduction studies are consistent with an isolated complete injury of the common peroneal nerve. Which of the following deficits is most likely on physical examination? A) Dorsiflexion of ankle B) Plantarflexion of great toe C) Sensation of lateral foot D) Sensation of medial foot E) Sensation of plantar foot
The correct response is Option A.
The common peroneal nerve forms as the sciatic nerve bifurcates at the apex of the popliteal fossa. It then follows the medial border of the biceps femoris muscle and tendon. The nerve then passes over the posterior aspect of the fibular head and winds around the neck of the fibula. The common peroneal then divides into the deep and superficial peroneal nerve branches. The deep branch supplies the anterior muscles of the leg, the dorsum of the foot, and the skin of the first web space. The superficial branch supplies the peroneus longus and brevis muscles and the skin on the distal third of the lower leg and dorsum of the foot. Because of its relatively superficial position, the common peroneal nerve is the most commonly injured nerve of the lower extremity. Transection of the common peroneal nerve results in paralysis of all muscles in the anterior and lateral compartments of the leg (dorsiflexors and ankle evertors). This pattern of injury results in the classic picture of a foot drop. The distribution of sensory loss would include the anterolateral leg and dorsum of the foot.
Sensation of the medial foot is from the saphenous nerve and branches of the medial plantar nerve. Lateral foot sensation is provided by the sural nerve. Sensation of the plantar aspect of the foot is from the terminal branches of the tibial nerve (medial and lateral plantar nerves). All of the muscles of plantar flexion of the ankle and toes (i.e. gastrocnemius, soleus, plantaris, and tibialis posterior, flexor hallucis longus, flexor digitorum longus, and the intrinsic plantar foot muscles) are innervated by the tibial nerve.
A 26-year-old man sustained a crush injury to the tip of the left middle finger with an associated fracture at the dorsal base of the distal phalanx with nail bed injury 6 months ago. No treatment was provided. Examination shows non-union of the distal phalanx. Which of the following is the most likely secondary deformity in this patient? A) Boutonniere deformity B) Jersey finger C ) Quadriga D) Swan neck deformity E)Trigger finger
The correct response is Option D.
The scenario described involves a bony mallet deformity in which a distal phalanx fracture is associated with disruption of terminal extension at the distal interphalangeal joint. If untreated, the DIP extension loss due to a non-union of a bony mallet injury may progress to a swan neck deformity through compensatory proximal phalangeal hyperextension in the setting of continued and persistent flexion at the distal interphalangeal joint (from unopposed pull of the flexor digitorum profundus tendon). A secondary swan neck deformity may occur because of dorsal subluxation of the lateral bands and attenuation of the volar plate and transverse retinacular ligament at the PIP joint level.
A jersey finger is caused by rupture of the terminal flexor digitorum profundus. A boutonniere deformity can be caused by an injury to the central slip (but not the terminal extensor tendon). Quadriga is due to loss of length of a repaired FDP tendon, causing the finger with the repaired tendon to reach terminal flexion sooner than the other fingers whose FDP tendons are of normal length. A trigger finger does not involve a fracture of the DIP joint.
A 22-year-old man who is a college student sustains a volar oblique fingertip amputation while chopping vegetables. Examination shows involvement of the hyponychium, but the nail is undamaged. The wound measures 1 × 1.5 cm, and no exposed bone is noted. Which of the following is the most appropriate treatment to encourage healing by secondary intention?
A) Apply negative pressure wound therapy
B) Apply povidone iodine to the wound daily and cover with dry gauze
C) Cover wound with semiocclusive dressing and change weekly
D) Leave wound open to air
E) Soak wound in hydrogen peroxide daily and cover with moist gauze
Fingertip or thumb tip amputations that result in small wounds (less than 1.5 cm2) and minimal exposed bone are best managed with healing by secondary intention. The only exception to this might be a laborer anxious to get back to work with a healed wound sooner than 3 to 4 weeks. Mennen reported a series of 200 such injuries treated with a semi-occlusive dressing, and average healing time was 20 days.
A semiocclusive dressing is semi-permeable and transparent, allowing air to pass through the dressing, but providing a barrier to moisture. Commonly available semipermeable dressings are marketed under brand names like Tegaderm (3M) and OPSITE (Smith & Nephew). These dressings maintain a moist wound environment, which speeds healing. If dressings are changed every 5 to 7 days, manipulation of the wound is minimized and, therefore, healing is less disrupted.
Leaving a wound open to air would allow tissues to dry out, which would delay
healing. Likewise, the use of povidone-iodine and/or hydrogen peroxide would slow down healing due to drying of the wound. Although these topical agents are effective at eliminating bacteria from dirty or infected wounds, prolonged use will interfere with normal wound healing. Finally, a wound of this small size would not warrant negative pressure wound therapy. Even the small, intrinsically-powered negative pressure wound therapy devices would not offer any advantages over a semiocclusive dressing and would increase cost substantially.
When a pedicled sural flap is raised to the heel, which of the following is the origin of the arterial blood supply? A) Descending genicular artery B) Lateral sural artery C) Medial femoral circumflex artery D) Medial plantar artery E) Peroneal artery
The correct response is Option E.
The reverse sural flap is a fasciocutaneous flap often used for ankle or heel wounds. The blood supply of the flap can be from a median superficial artery or the arterial plexus that travels with the sural nerve; the origin is a lower peroneal perforator located approximately 5 cm proximal to the lateral malleolus.
The lateral sural artery would be the appropriate blood supply for perfusion of a pedicled lateral gastrocnemius flap. The gracilis flap blood supply derives from the medial circumflex artery. The descending genicular artery provides the blood supply of the medial femoral condyle flap. The medial plantar artery is the blood supply for the medial plantar artery flap.
A 44-year-old man presents with a 20-degree extension deformity of his wrist because of burn scarring. Z-plasty for contracture release is planned. Which of the following (A-D) is the most appropriate placement of the central limb for this procedure in this patient?
The correct response is Option A.
Z-plasty is a fundamental and common reconstructive technique used to elongate scars or contractures, narrow scars, rearrange tissues into relaxed skin tension lines, camouflage scars, or releasing tension. This is a form of rotation and advancement whereby a central limb of the Z-plasty is drawn parallel to the line of maximal tension, and subsequent limbs are drawn anywhere from 30 to 90 degrees from this. Wider angles give greater scar elongation at the expense of greater transverse tension.
In this scenario, the line of maximum tension is longitudinal, creating an extension deformity. Maximal contracture release will occur with a central limb of the Z-plasty drawn longitudinally.
A 42-year-old man presents with a painful subungual glomus tumor of the index finger. Definitive treatment should consist of which of the following? A) Chemotherapy B) Complete surgical excision C) External beam radiation therapy D) Immunotherapy E) Intralesional steroid injection
The correct response is Option B.
Glomus tumors include approximately 1 to 5% of soft tissue tumors of the hand. The majority are subungual. Presentation is typically a raised blue or pink nodule that can discolor or deform the nail. Love’s pin test is performed by applying pressure to the area with a pinhead, causing exquisite pain. Diagnosis can be aided with plain film and MRI. Treatment includes complete surgical excision. There is no role for radiation, chemotherapy, immunotherapy, or steroid injections in definitive treatment.
A 37-year-old woman presents for evaluation of a laceration to the mid humerus that she sustained in a motor vehicle collision. On examination, the patient is unable to extend the wrist, fingers, or thumb. Surgical exploration shows complete radial nerve transection; the median/ulnar nerves are intact. Direct neurorrhaphy is performed after debridement and mobilization of the nerve ends. Which of the following is the last muscle to be reinnervated during nerve recovery? A) Abductor pollicis brevis B) Abductor pollicis longus C) Brachioradialis D) Extensor carpi radialis brevis E) Extensor indicis proprius
The correct response is Option E.
Radial nerve injuries may occur in the setting of humeral fractures, and transection is most common in the setting of an open injury. The most important components of functional recovery following radial nerve injury include wrist, finger, and thumb extension. The order of reinnervation of the radial-innervated muscles is most commonly brachioradialis, extensor carpi radialis longus, supinator, extensor carpi radialis brevis, extensor digitorum communis, extensor carpi ulnaris, extensor digiti quinti, abductor pollicis longus, extensor pollicis longus, extensor pollicis brevis, and extensor indicis proprius. The abductor pollicis brevis is innervated by the median nerve via the thenar motor branch.